Neuro Exam 4

Pataasin ang iyong marka sa homework at exams ngayon gamit ang Quizwiz!

Which of the following assessment data indicated nuchal rigidity? A. Positive Kernig's sign B. Negative Brudzinski's sign C. Positive homan's sign D. Negative Kernig's sign

A. Positive Kernig's sign Explanation: A positive Kernig's sign indicated nuchal rigidity, caused by an irritative lesion of the subarachnoid space. Brudzinski's sign is also indicative of the condition.

A client with trigeminal neuralgia is being treated with carbamazepine (Tegretol), 400 mg orally daily. Which value indicates that the client is experiencing an adverse effect to the medication? 1. Uric acid level, 5 mg/dL 2. Sodium level, 140 mEq/L 3. Blood urea nitrogen level, 15 mg/dL 4. White blood cell count, 3000 cells/mm3

4. White blood cell count, 3000 cells/mm3 Rationale: Adverse effects of carbamazepine appear as blood dyscrasias, including aplastic anemia, agranulocytosis, thrombocytopenia, and leukopenia; cardiovascular disturbances including thrombophlebitis and dysrhythmias; and dermatological effects. The low white blood cell count reflects agranulocytosis.

A male client has a history of painful, continuous muscle spasms. He has taken several skeletal muscle relaxants without experiencing relief. His physician prescribes diazepam (Valium), 2 mg P.O. twice daily. In addition to being used to relieve painful muscle spasms, diazepam also is recommended for: A. long-term treatment of epilepsy. B. postoperative pain management of laminectomy clients. C. postoperative pain management of diskectomy clients D. treatment of spasticity associated with spinal cord lesions.

D. treatment of spasticity associated with spinal cord lesions. Explanation: In addition to relieving painful muscle spasms, diazepam also is recommended for treatment of spasticity associated with spinal cord lesions. Diazepam's use is limited by its central nervous system effects and the tolerance that develops with prolonged use. The parenteral form of diazepam can treat status epilepticus, but the drug's sedating properties make it an unsuitable choice for long-term management of epilepsy. Diazepam isn't an analgesic agent.

The nurse is providing care to a client with increased intracranial pressure (ICP). Which approach is beneficial in controlling the client's ICP from an environmental viewpoint? 1. Reduce environmental noise. 2. Allow visitors as desired by the client and family. 3. Cluster nursing activities to reduce the number of interruptions. 4. Awaken the client every 2 to 3 hours to monitor mental status.

1. Reduce environmental noise. Rationale: Nursing interventions to control the ICP include maintaining a calm, quiet, and restful environment. Environmental noise should be kept at a minimum. Visiting should be monitored to avoid emotional stress and interruption of sleep. Interventions should be spaced out over the shift to minimize the risk of a sustained rise in ICP.

A client who is experiencing an inferior wall myocardial infarction has had a drop in heart rate into the 50 to 56 beats/min range. The client is also complaining of nausea. The nurse interprets that these symptoms are because of stimulation of which cranial nerve (CN)? 1. Vagus (CN X) 2. Hypoglossal (CN XII) 3. Spinal accessory (CN XI) 4. Glossopharyngeal (CN IX)

1. Vagus (CN X) Rationale: The vagus nerve is responsible for sensations in the thoracic and abdominal viscera. It is also responsible for the decrease in heart rate because approximately 75% of all parasympathetic stimulation is carried by the vagus nerve. CN IX is responsible for taste in the posterior two thirds of the tongue, pharyngeal sensation, and swallowing. CN XI is responsible for neck and shoulder movement. CN XII is responsible for tongue movement. Source: IGGY

The nurse is assessing the function of cranial nerve XII in a client who sustained a stroke. To assess function of this nerve, which action should the nurse ask the client to perform? 1. Extend the arms. 2. Extend the tongue. 3. Turn the head toward the nurse's arm. 4. Focus the eyes on the object held by the nurse

2. Extend the tongue. Rationale: Impairment of cranial nerve XII can occur with a stroke. To assess the function of cranial nerve XII (the hypoglossal nerve), the nurse would assess the client's ability to extend the tongue.

A client reports frequent use of acetaminophen (Tylenol) for relief of headaches and other discomforts. The nurse should evaluate which diagnostic data to determine if the client is at risk for toxicity? 1. Chest x-ray 2. The electrocardiogram 3. The liver function studies 4. The upper gastrointestinal x-ray results

3. The liver function studies Rationale: In adults, overdose of acetaminophen causes liver damage. In addition, clients with liver disorders are at higher risk of experiencing hepatotoxicity with chronic acetaminophen use.

A nurse is reading the laboratory results for a client being treated with carbamazepine (Tegretol) for prophylaxis of complex partial seizures. When evaluating the client's laboratory data, the nurse determines that which value is consistent with an adverse effect of this medication? 1. Sodium level, 136 mEq/L 2. Platelet count, 350,000 cells/mm3 3. White blood cell count, 3200 cells/mm3 4. Blood urea nitrogen (BUN), 19 mg/dL

3. White blood cell count, 3200 cells/mm3 Rationale: Adverse effects of carbamazepine appear as blood dyscrasias, including aplastic anemia, agranulocytosis, thrombocytopenia, and leukopenia. Other adverse effects include cardiovascular disturbances, thrombophlebitis, dysrhythmias, and dermatological effects.

A client has been prescribed cyclobenzaprine (Flexeril) for the treatment of painful muscle spasms accompanying a herniated intervertebral disk. The nurse should withhold the medication and question the prescription if the client has a concurrent prescription for which medication? 1. Furosemide (Lasix) 2. Ibuprofen (Motrin IB) 3. Valproic acid (Depakene) 4. Tranylcypromine (Parnate)

4. Tranylcypromine (Parnate) Rationale: The client should not receive cyclobenzaprine if the client has taken monoamine oxidase inhibitors such as tranylcypromine or phenelzine within the last 14 days. Otherwise, the client could experience hyperpyretic crisis, seizures, and possibly death. The medications in the remaining options are not contraindicated.

A client with a spinal cord injury at the level of C5 has a weakened respiratory effort and ineffective cough and is using accessory neck muscles in breathing. The nurse carefully monitors the client and suspects the presence of which complication? 1. Altered breathing pattern 2. Increased likelihood of injury 3. Ineffective oxygen consumption 4. Increased susceptibility to aspiration

1. Altered breathing pattern Rationale: Altered breathing pattern indicates that the respiratory rate, depth, rhythm, timing, or chest wall movements are insufficient for optimal ventilation of the client. This is a risk for clients with spinal cord injury in the lower cervical area. Ineffective oxygen consumption occurs when oxygenation or carbon dioxide elimination is altered at the alveolar-capillary membrane. Increased susceptibility to aspiration and increased likelihood of injury are unrelated to the focus of the question.

The nurse in the neurological unit is monitoring a client for signs of increased intracranial pressure (ICP). The nurse reviews the assessment findings for the client and notes documentation of the presence of Cushing's reflex. The nurse determines that the presence of this reflex is obtained by assessing which item? 1. Blood pressure 2. Motor response 3. Pupillary response 4. Level of consciousness

1. Blood pressure Rationale: Cushing's reflex is a late sign of increased ICP and consists of a widening pulse pressure (systolic pressure rises faster than diastolic pressure) and bradycardia. Options 2, 3, and 4 are unrelated to monitoring for Cushing's reflex.

A client is admitted to the hospital with a diagnosis of neurogenic shock after a traumatic motor vehicle collision. Which manifestation best characterizes this diagnosis? 1. Bradycardia 2. Hyperthermia 3. Hypoglycemia 4. Increased cardiac output

1. Bradycardia Rationale: Neurogenic shock can occur after a spinal cord injury. Usually the body attempts to compensate massive vasodilation by becoming tachycardic to increase the amount of blood flow and oxygen delivered to the tissues; however, in neurogenic shock, the sympathetic nervous system is disrupted, so the parasympathetic system takes over, resulting in bradycardia. This insufficient pumping of the heart leads to a decrease in cardiac output. Hypothermia develops because of the vasodilation and the inability to control body temperature through vasoconstriction. Source: IGGY

A health care provider in the emergency department prescribes a thrombolytic medication for a client with an acute ischemic stroke. His wife asks the nurse how the medication works. How should the nurse respond about how the medication will work? 1. Dissolve clots. 2. Prevent ischemia. 3. Prevent bleeding. 4. Decrease the client's anxiety.

1. Dissolve clots Rationale: Thrombolytic medications are used to treat acute thrombolytic disorders. These medications dissolve clots. Because these medications alter the hemostatic capability of the client, any bleeding that does occur can be difficult to control. Source: IGGY and Kee Hayes

The nurse is performing a neurological assessment on a client and is assessing the function of cranial nerves III, IV, and VI. Assessment of which aspect of function by the nurse will yield the best information about these cranial nerves? 1. Eye movements 2. Response to verbal stimuli 3. Affect, feelings, or emotions 4. Insight, judgment, and planning

1. Eye movements Rationale: Eye movements are under the control of cranial nerves III, IV, and VI. Level of consciousness (response to verbal stimuli) is controlled by the reticular activating system and both cerebral hemispheres. Feelings are part of the role of the limbic system and involve both hemispheres. Insight, judgment, and planning are part of the function of the frontal lobe in conjunction with association fibers that connect to other areas of the cerebrum.

The nurse has the following prescription for a postcraniotomy client, "dexamethasone (Decadron) 4 mg by the intravenous (IV) route now." How does the nurse administer the medication? 1. IV push over 1 minute 2. IV push over 4 minutes 3. IV piggyback in 50 mL of normal saline over 10 minutes 4. IV piggyback in 50 mL of normal saline over 30 minutes

1. IV push over 1 minute Rationale: Dexamethasone (Decadron) is an adrenocorticosteroid administered after craniotomy to control cerebral edema. It is given by IV push, and single doses are administered over 1 minute. Dexamethasone IV doses are changed to the oral route after 24 to 72 hours and are tapered until discontinued. Additionally, IV fluids are administered cautiously after craniotomy to prevent increased cerebral edema. Source: IGGY

The client with a head injury is experiencing signs of increased intracranial pressure (ICP), and mannitol (Osmitrol) is prescribed. The nurse administering this medication expects which as an intended effects of this medication? Select all that apply. 1. Increased diuresis 2. Reduced intracranial pressure 3. Increased osmotic pressure of glomerular filtrate 4. Reduced tubular reabsorption of water and solutes 5. Reabsorption of sodium and water in the loop of Henle

1. Increased diuresis 2. Reduced intracranial pressure 3. Increased osmotic pressure of glomerular filtrate 4. Reduced tubular reabsorption of water and solutes Rationale: Mannitol is an osmotic diuretic that induces diuresis by raising the osmotic pressure of glomerular filtrate, thereby inhibiting tubular reabsorption of water and solutes. It is used to reduce intracranial pressure in the client with head trauma. The incorrect option would cause fluid retention through reabsorption, thereby increasing the intracranial pressure.

The nurse is caring for a client with an exacerbation of multiple sclerosis. Which medication(s) will the nurse expect to be prescribed to hasten recovery from the exacerbation? 1. Methylprednisolone (Solu-Medrol) intravenously 2. Carbamazepine (Tegretol) and phenytoin (Dilantin) by mouth 3. Phenytoin (Dilantin) intravenously, then tapered to oral route 4. Lioresal (Baclofen) by mouth and diazepam (Valium) intravenously

1. Methylprednisolone (Solu-Medrol) intravenously Rationale: Intravenous methylprednisolone or adrenocorticotropic hormone may be prescribed to accelerate recovery from an exacerbation of multiple sclerosis. Carbamazepine may be prescribed for trigeminal neuralgia, and phenytoin may be prescribed to control seizures. Lioresal and diazepam are used to treat muscle spasticity.

Which interventions would be included in the care of a client with a head injury and a subarachnoid bolt? Select all that apply. 1. Monitor vital signs. 2. Monitor neurological status. 3. Monitor the dressing for signs of infection. 4. Monitor for signs of increased intracranial pressure. 5. Drain cerebrospinal fluid when the intracranial pressure is elevated.

1. Monitor vital signs. 2. Monitor neurological status. 3. Monitor the dressing for signs of infection. 4. Monitor for signs of increased intracranial pressure. Rationale: A subarachnoid bolt is inserted into the subarachnoid space and is used to measure intracranial pressure. Because a subarachnoid bolt is placed in the subarachnoid space, it is not capable of draining cerebrospinal fluid, which is produced in the ventricles. Therefore option 4 is not an intervention. Source: IGGY

The nurse is assigned to care for a client with complete right-sided hemiparesis. Which characteristics are associated with this condition? Select all that apply. 1. The client is aphasic. 2. The client has weakness in the face and tongue. 3. The client has weakness on the right side of the body. 4. The client has complete bilateral paralysis of the arms and legs. 5. The client has lost the ability to move the right arm but is able to walk independently. 6. The client has lost the ability to ambulate independently but is able to feed and bathe himself or herself without assistance.

1. The client is aphasic. 2. The client has weakness in the face and tongue. 3. The client has weakness on the right side of the body. Rationale: Hemiparesis is a weakness of one side of the body that may occur after a stroke. It involves weakness of the face and tongue, arm, and leg on one side. These clients are also aphasic: unable to discriminate words and letters. They are generally very cautious and get anxious when attempting a new task. Complete bilateral paralysis does not occur in this hemiparesis. The client with right-sided hemiparesis has weakness of the right arm and leg and needs assistance with feeding, bathing, and ambulating. Source: IGGY

The nurse is developing a plan of care for a client with dysphagia following a stroke (brain attack). Which should the nurse include in the plan? Select all that apply. 1. Thicken liquids. 2. Assist the client with eating. 3. Assess for the presence of a swallow reflex. 4. Place the food on the affected side of the mouth. 5. Provide ample time for the client to chew and swallow.

1. Thicken liquids. 2. Assist the client with eating. 3. Assess for the presence of a swallow reflex. 5. Provide ample time for the client to chew and swallow. Rationale: Liquids are thickened to prevent aspiration. The nurse should assist the client with eating and place food on the unaffected side of the mouth. The nurse should assess for gag and swallowing reflexes before the client with dysphagia is started on a diet. The client should be allowed ample time to chew and swallow to prevent choking. Source: IGGY

A client has a cerebellar lesion. The nurse determines that the client is adapting successfully to this problem if the client demonstrates proper use of which item? 1. Walker 2. Slider board 3. Raised toilet seat 4. Adaptive eating utensils

1. Walker Rationale: The cerebellum is responsible for balance and coordination. A walker would provide stability for the client during ambulation. Adaptive eating utensils may be beneficial if the client has partial paralysis of the hand. A raised toilet seat is useful if the client does not have the mobility or ability to flex the hips. A slider board is used in transferring a client from a bed to a stretcher or wheelchair. Source: IGGY

A client with a traumatic closed head injury shows signs that indicate the presence of cerebral edema. Which intravenous solution would increase cellular swelling and cerebral edema? 1. 0.9% normal saline 2. 0.45% normal saline 3. 5% dextrose in water 4. Lactated Ringer's solution

2. 0.45% normal saline Rationale: Hypotonic solutions such as 0.45% normal saline are inappropriate for the client with cerebral edema because hypotonic solutions have the potential to cause cellular swelling and cerebral edema. The remaining choices of solutions would be appropriate because they are examples of isotonic solutions and thus are similar in composition to plasma. These fluids would remain in the intravascular space without potentiating the client's cerebral edema.

The nurse is preparing to care for a client who has undergone myelography using an oil-based contrast agent. How long and in what position should the nurse plan to position the client on bed rest? 1. 2 hours, with the head of bed flat 2. 8 hours, with the head of bed flat 3. 4 hours, with head of bed elevated 15 to 30 degrees 4. 8 hours, with head of bed elevated 15 to 30 degrees

2. 8 hours, with the head of bed flat Rationale: If an oil-based dye is used during myelography, the dye is removed at the end of the procedure. The client is positioned flat in bed for approximately 8 hours after the dye is removed. If a water-based contrast medium is used, the client is positioned with the head of bed elevated 30 degrees for 6 to 8 hours to keep the dye from irritating the cerebral meninges.

A client has a neurological deficit involving the limbic system. Which assessment finding is specific to this type of deficit? 1. Is disoriented to person, place, and time 2. Affect is flat, with periods of emotional lability 3. Cannot recall what was eaten for breakfast today 4. Demonstrates inability to add and subtract; does not know who is the president of the United States

2. Affect is flat, with periods of emotional lability Rationale: The limbic system is responsible for feelings (affect) and emotions. Calculation ability and knowledge of current events relate to function of the frontal lobe. The cerebral hemispheres, with specific regional functions, control orientation. Recall of recent events is controlled by the hippocampus. Source: IGGY

The nurse is assessing a client's gait, which is characterized by unsteadiness and staggering steps. The nurse determines the presence of which type of gait? 1. Spastic 2. Ataxic 3. Festinating 4. Dystrophic or broad-based

2. Ataxic Rationale: An ataxic gait is characterized by unsteadiness and staggering.

A nurse has a prescription to administer a medication to a client who is experiencing shivering as a result of hyperthermia. Which medication should the nurse anticipate to be prescribed? 1. Buspirone (BuSpar) 2. Chlorpromazine (Thorazine) 3. Prochlorperazine (Compazine) 4. Fluphenazine (Prolixin Decanoate)

2. Chlorpromazine (Thorazine) Rationale: Chlorpromazine is used to control shivering in hyperthermic states. It is a phenothiazine and has antiemetic and antipsychotic uses, especially when psychosis is accompanied by increased psychomotor activity. Source: IGGY

The nurse is caring for the client with increased intracranial pressure. The nurse would note which trend in vital signs if the intracranial pressure is rising? 1. Increasing temperature, increasing pulse, increasing respirations, decreasing blood pressure 2. Increasing temperature, decreasing pulse, decreasing respirations, increasing blood pressure 3. Decreasing temperature, decreasing pulse, increasing respirations, decreasing blood pressure 4. Decreasing temperature, increasing pulse, decreasing respirations, increasing blood pressure

2. Increasing temperature, decreasing pulse, decreasing respirations, increasing blood pressure Rationale: A change in vital signs may be a late sign of increased intracranial pressure. Trends include increasing temperature and blood pressure and decreasing pulse and respirations. Respiratory irregularities also may occur. Source: IGGY

The home care nurse is making a visit to a client who is wheelchair bound after a spinal cord injury sustained 4 months earlier. Just before leaving the home, the nurse ensures that which intervention has been done to prevent an episode of autonomic dysreflexia (hyperreflexia)? 1. Updating the home safety sheet 2. Leaving the client in an unchilled area of the room 3. Noting a bowel movement on the client progress note 4. Recording the amount of urine obtained with catheterization

2. Leaving the client in an unchilled area of the room Rationale: The most common cause of autonomic dysreflexia is visceral stimuli, such as with blockage of urinary drainage or with constipation. Barring these, other causes include noxious mechanical and thermal stimuli, particularly pressure and overchilling. For this reason, the nurse ensures that the client is positioned with no pinching or pressure on paralyzed body parts and that the client will be sufficiently warm. Source: IGGY

The nurse in the health care clinic is providing medication instructions to a client with a seizure disorder who will be taking divalproex sodium (Depakote). The nurse should instruct the client about the importance of returning to the clinic for monitoring of which laboratory study? 1. Electrolyte panel 2. Liver function studies 3. Renal function studies 4. Blood glucose level determination

2. Liver function studies Rationale: Divalproex sodium, an anticonvulsant, can cause fatal hepatotoxicity. The nurse should instruct the client about the importance of monitoring the results of liver function studies and ammonia level determinations.

The emergency department nurse is caring for a client with a suspected diagnosis of meningitis. The nurse should prepare the client for which test to confirm the diagnosis? 1. Blood culture 2. Lumbar puncture 3. Serum electrolyte panel 4. White blood cell (WBC) count

2. Lumbar puncture Rationale: Meningitis is an acute or a chronic inflammation of the meninges and the cerebrospinal fluid (CSF). The most significant diagnostic test used in meningitis is the lumbar puncture. Clients older than 60 years, those who are immunocompromised, or those who have signs of increased intracranial pressure usually have a computed tomography scan before the lumbar puncture. Blood cultures are not normally prescribed for diagnosis of this disorder; however, if there will be a delay in obtaining the CSF, blood may be drawn for culture and sensitivity. A WBC count and serum electrolyte assay also may be performed but will not confirm the diagnosis. Source: IGGY

The nurse is assessing the motor function of an unconscious client. The nurse should plan to use which technique to test the client's peripheral response to pain? 1. Sternal rub 2. Nail bed pressure 3. Pressure on the orbital rim 4. Squeezing of the sternocleidomastoid muscle

2. Nail bed pressure Rationale: Motor testing in the unconscious client can be done only by testing response to painful stimuli. Nail bed pressure tests a basic peripheral response. Source:

A client with a neurological deficit is able, with eyes closed, to identify a set of keys placed in his or her hands. A nurse observing the client interprets this to mean that which area of the brain is intact? 1. Frontal lobe 2. Parietal lobe 3. Temporal lobe 4. Occipital lobe

2. Parietal lobe Rationale: The ability to distinguish an object by touch is called stereognosis, which is a function of the right parietal area. The parietal lobe of the brain is responsible for spatial orientation and awareness of sizes and shapes. The left parietal area is responsible for mathematics and right-left orientation. The other lobes of the brain are not responsible for this function. Source: IGGY

The nurse develops a plan of care for a client with a brain aneurysm who will be placed on aneurysm precautions. Which interventions should be included in the plan? Select all that apply. 1. Leave the lights on in the client's room at night. 2. Place a blood pressure cuff at the client's bedside. 3. Close the shades in the client's room during the day. 4. Allow the client to drink one cup of caffeinated coffee a day. 5. Allow the client to ambulate four times a day with assistance.

2. Place a blood pressure cuff at the client's bedside. 3. Close the shades in the client's room during the day. Rationale: Aneurysm precautions include placing the client on bed rest in a quiet setting. The use of lights is kept to a minimum to prevent environmental stimulation. The nurse should monitor the blood pressure and note any changes that could indicate rupture. Any activity, such as pushing, pulling, sneezing, or straining, that increases the blood pressure or impedes venous return from the brain is prohibited. The nurse provides physical care to minimize increases in blood pressure. Visitors, radio, television, and reading materials are restricted or limited. Stimulants, such as nicotine and coffee and other caffeine-containing products, are prohibited. Decaffeinated coffee or tea may be used. Source: IGGY

The nurse is reviewing the laboratory results from a lumbar puncture performed in a client with a diagnosis of meningitis. Which laboratory findings are expected to be noted with bacterial meningitis? Select all that apply. 1. Increased glucose level 2. Protein level of 20 mg/dL 3. Increased white blood cells 4. Clear appearance of the cerebrospinal fluid 5. Cerebrospinal fluid (CSF) pressure of 250 mm H2O

2. Protein level of 20 mg/dL 3. Increased white blood cells 5. Cerebrospinal fluid (CSF) pressure of 250 mm H2O Rationale: If a bacterial infection of CSF is present, findings include reduced glucose level, a protein level greater than 15 mg/dL, increased white blood cells, a cloudy appearance of CSF, and CSF pressure greater than 200 mm H2O Source: IGGY

The student nurse develops a plan of care for a client after a lumbar puncture. The nursing instructor corrects the student if the student documents which incorrect intervention in the plan? 1. Maintain the client in a flat position. 2. Restrict fluid intake for a period of 2 hours. 3. Assess the client's ability to void and move the extremities. 4. Inspect the puncture site for swelling, redness, and drainage.

2. Restrict fluid intake for a period of 2 hours. Rationale: After the lumbar puncture the client remains flat in bed for at least 2 hours, depending on the health care provider's prescriptions. A liberal fluid intake is encouraged to replace the cerebrospinal fluid removed during the procedure, unless contraindicated by the client's condition. The nurse checks the puncture site for redness and drainage and assesses the client's ability to void and move the extremities. Source: Pagana, Pagana

A client with neck and upper extremity pain has been diagnosed with cervical radiculitis. The nurse understands that the client's symptoms must be caused by pressure on which structures of the vertebral column? 1. Spinous process 2. Spinal nerve root 3. Central spinal cord 4. Posterior facet joints

2. Spinal nerve root Rationale: Radiculitis is a term used to describe spinal nerve root compression at the intervertebral foramen. Radiculitis can be caused by a number of factors, such as whiplash or ruptured intervertebral disk. In many cases, it is caused by malalignment that occurs with degenerative disease or bone spur formation. Source: IGGY

The nurse is performing an assessment on a client admitted to the nursing unit with a diagnosis of stroke (brain attack). On assessment, the nurse notes that the client is unable to understand spoken language. The nurse plans care, understanding that the client is experiencing impairment of which areas? 1. The occipital lobe 2. The auditory association areas 3. The frontal lobe and optic nerve tracts 4. Concept formation and abstraction areas

2. The auditory association areas Rationale: Auditory association and storage areas are located in the temporal lobe and relate to understanding spoken language. The occipital lobe contains areas related to vision. The frontal lobe controls voluntary muscle activity, including speech, and an impairment can result in expressive aphasia. The parietal lobe contains association areas for concept formation, abstraction, spatial orientation, body and object size and shape, and tactile sensation. Source: IGGY

Meperidine hydrochloride (Demerol) has been prescribed for a client to treat pain. Which are side/adverse effects of this medication? Select all that apply. 1. Diarrhea 2. Tremors 3. Drowsiness 4. Hypotension 5. Urinary frequency 6. Increased respiratory rate

2. Tremors 3. Drowsiness 4. Hypotension Rationale: Meperidine hydrochloride is an opioid analgesic. Side/adverse effects include respiratory depression, drowsiness, hypotension, constipation, urinary retention, nausea, vomiting, and tremors.

The home care nurse is performing an assessment on a client with a diagnosis of Bell's palsy. Which assessment question will elicit the most specific information regarding this client's disorder? 1. "Do your eyes feel dry?" 2. "Do you have any spasms in your throat?" 3. "Are you having any difficulty chewing food?" 4. "Do you have any tingling sensations around your mouth?"

3. "Are you having any difficulty chewing food?" Rationale: Bell's palsy is a one-sided facial paralysis caused by compression of the facial nerve. Manifestations include facial droop from paralysis of the facial muscles; increased lacrimation; painful sensations in the eye, face, or behind the ear; and speech or chewing difficulties. Source: IGGY

The nurse has given suggestions to a client with trigeminal neuralgia about strategies to minimize episodes of pain. The nurse determines that the client needs further education if the client makes which statement? 1. "I will wash my face with cotton pads." 2. "I'll have to start chewing on my unaffected side." 3. "I'll try to eat my food either very warm or very cold." 4. "I should rinse my mouth if toothbrushing is painful."

3. "I'll try to eat my food either very warm or very cold." Rationale: Facial pain can be minimized by using cotton pads to wash the face and using room temperature water. The client should chew on the unaffected side of the mouth, eat a soft diet, and take in foods and beverages at room temperature. If toothbrushing triggers pain, an oral rinse after meals may be helpful instead. Source: IGGY

A client is being scheduled for a positron emission tomography (PET) scan of the brain. The nurse should provide which explanation to the client? 1. "The test uses magnetic fields to produce images." 2. "The test provides cross-sectional views of the brain." 3. "The test detects abnormal glucose metabolism in the brain." 4. "The test views bones of the skull, nasal sinuses, and vertebrae."

3. "The test detects abnormal glucose metabolism in the brain." Rationale: The PET scan can detect abnormal brain tissue metabolism. A radionuclide is attached to a glucose component and is injected as an intravenous bolus. The computer records the chemical activity in the brain following injection Source: Pagana, Pagana

A client has dysfunction of the cochlear division of the vestibulocochlear nerve (cranial nerve VIII). The nurse should determine that the client is adequately adapting to this problem if he or she states a plan to obtain which item? 1. A walker 2. Eyeglasses 3. A hearing aid 4. A bath thermometer

3. A hearing aid Rationale: The cochlear division of cranial nerve VIII is responsible for hearing. Clients with hearing difficulty may benefit from the use of a hearing aid. Source: IGGY

The nurse is evaluating the status of a client who had a craniotomy 3 days ago. Which assessment finding would indicate that the client is developing meningitis as a complication of surgery? 1. A negative Kernig sign 2. Absence of nuchal rigidity 3. A positive Brudzinski sign 4. A Glasgow Coma Scale score of 15

3. A positive Brudzinski sign Rationale: Signs of meningeal irritation compatible with meningitis include nuchal rigidity, a positive Brudzinski sign, and positive Kernig sign. Nuchal rigidity is characterized by a stiff neck and soreness, which is especially noticeable when the neck is flexed. Kernig's sign is positive when the client feels pain and spasm of the hamstring muscles when the leg is fully flexed at the knee and hip. Brudzinski's sign is positive when the client flexes the hips and knees in response to the nurse gently flexing the head and neck onto the chest. A Glasgow Coma Scale score of 15 is a perfect score and indicates that the client is awake and alert, with no neurological deficits.

Dantrolene sodium (Dantrium) has been administered to a client with a spinal cord injury. The nurse determines that the client is experiencing an adverse effect of the medication if which is noted? 1. Dizziness 2. Drowsiness 3. Abdominal pain 4. Lightheadedness

3. Abdominal pain Rationale: Dantrium is hepatotoxic. The nurse observes for indications of liver dysfunction, which include jaundice, abdominal pain, and malaise. The nurse notifies the health care provider if these occur. The items in options 1, 2, and 4 are expected side effects due to the central nervous system (CNS)-depressant effects of the medication. Source: Kee Hayes

The family of a client with a spinal cord injury rushes to the nursing station, saying that the client needs immediate help. On entering the room, the nurse notes that the client is diaphoretic with a flushed face and neck and is complaining of a severe headache. The pulse rate is 40 beats/minute and the blood pressure is 230/100 mm Hg. The nurse acts quickly, suspecting that the client is experiencing which condition? 1. Spinal shock 2. Pulmonary embolism 3. Autonomic dysreflexia 4. Malignant hyperthermia

3. Autonomic dysreflexia Rationale: The client with a spinal cord injury is at risk for autonomic dysreflexia with an injury above the level of the seventh thoracic vertebra (T7). Autonomic dysreflexia is characterized by severe, throbbing headache; flushing of the face and neck; bradycardia; and sudden severe hypertension. Other signs include nasal stuffiness, blurred vision, nausea, and sweating. Autonomic dysreflexia is a life-threatening syndrome triggered by a noxious stimulus below the level of the injury. Source: IGGY

The nurse is assessing a client with a neurological deficit involving the hippocampus. Which finding is indicative of this deficit? 1. Disoriented to client, place, and time 2. Affect flat, with periods of emotional lability 3. Cannot recall what was eaten for breakfast today 4. Unable to add and subtract; does not know who is president

3. Cannot recall what was eaten for breakfast today Rationale: Recall of recent events and the storage of memories are controlled by the hippocampus, which is a limbic system structure. The cerebral hemispheres, with specific regional functions, control orientation. The limbic system, overall, is responsible for feelings, affect, and emotions. Calculation ability and knowledge of current events are under the control of the frontal lobes of the cerebrum.

The nurse is administering medications to a client with trigeminal neuralgia. The nurse expects that which medication will be prescribed for pain relief? 1. Oxycodone plus aspirin (Percodan) 2. Acetaminophen (Tylenol) and codeine sulfate 3. Carbamazepine (Tegretol) and gabapentin (Neurontin) 4. Meperidine hydrochloride (Demerol) and hydroxyzine (Vistaril)

3. Carbamazepine (Tegretol) and gabapentin (Neurontin) Rationale: The anticonvulsant medications carbamazepine and gabapentin (Neurontin) help relieve the pain in many clients with trigeminal neuralgia. They act by inhibiting the reactivity of neurons in the trigeminal nerve. Opioid analgesics (meperidine hydrochloride, codeine sulfate, and oxycodone) are not very effective in controlling pain caused by trigeminal neuralgia. Source: IGGY

A lumbar puncture is performed on a child suspected to have bacterial meningitis, and cerebrospinal fluid (CSF) is obtained for analysis. The nurse reviews the results of the CSF analysis and determines that which results would verify the diagnosis? 1. Clear CSF, decreased pressure, and elevated protein level 2. Clear CSF, elevated protein, and decreased glucose levels 3. Cloudy CSF, elevated protein, and decreased glucose levels 4. Cloudy CSF, decreased protein, and decreased glucose levels

3. Cloudy CSF, elevated protein, and decreased glucose levels Rationale: Meningitis is an infectious process of the central nervous system caused by bacteria and viruses; it may be acquired as a primary disease or as a result of complications of neurosurgery, trauma, infection of the sinus or ears, or systemic infections. Meningitis is diagnosed by testing cerebrospinal fluid obtained by lumbar puncture. In the case of bacterial meningitis, findings usually include an elevated pressure; turbid or cloudy cerebrospinal fluid; and elevated leukocyte, elevated protein, and decreased glucose levels.

The nurse in the neurological unit is caring for a client who was in a motor vehicle crash and sustained a blunt head injury. On assessment of the client, the nurse notes the presence of bloody drainage from the nose. Which nursing action is most appropriate? 1. Insert nasal packing. 2. Document the findings. 3. Contact the health care provider (HCP). 4. Monitor the client's blood pressure and check for signs of increased intracranial pressure.

3. Contact the health care provider (HCP). Rationale: Bloody or clear drainage from either the nasal or the auditory canal after head trauma could indicate a cerebrospinal fluid leak. The appropriate nursing action is to notify the HCP, because this finding requires immediate intervention. Source: IGGY

A client with trigeminal neuralgia tells the nurse that acetaminophen (Tylenol) is taken daily for the relief of generalized discomfort. Which laboratory value would indicate toxicity associated with the medication? 1. Sodium level of 140 mEq/L 2. Prothrombin time of 11.8 seconds 3. Direct bilirubin level of 2 mg/dL 4. Platelet count of 400,000 cells/mm3

3. Direct bilirubin level of 2 mg/dL Rationale: In adults, overdose of acetaminophen causes liver damage. The correct option is an indicator of liver function and is the only option that indicates an abnormal laboratory value. The normal direct bilirubin level is 0 to 0.3 mg/dL.

The nurse is evaluating the neurological signs of a client in spinal shock following spinal cord injury. Which observation indicates that spinal shock persists? 1. Hyperreflexia 2. Positive reflexes 3. Flaccid paralysis 4. Reflex emptying of the bladder

3. Flaccid paralysis Rationale: Resolution of spinal shock is occurring when there is return of reflexes (especially flexors to noxious cutaneous stimuli), a state of hyperreflexia rather than flaccidity, and reflex emptying of the bladder.

The nurse is monitoring a client who has returned to the nursing unit after a myelogram. Which client complaint would indicate the need to notify the health care provider (HCP)? 1. Backache 2. Headache 3. Neck stiffness 4. Feelings of fatigue

3. Neck stiffness Rationale: Headache is relatively common after the procedure, but neck stiffness, especially on flexion, and pain should be reported because they signal meningeal irritation. The client also is monitored for evidence of allergic reactions to the dye such as confusion, dizziness, tremors, and hallucinations. Feelings of fatigue may be normal, and back discomfort may be owing to the positions required for the procedure. Source: Pagana, Pagana

A client is being hyperventilated by a mechanical ventilator to decrease the client's intracranial pressure (ICP). On monitoring arterial blood gas results, the nurse should expect values that are within which ranges? 1. Pao2 60 to 100 mm Hg, Paco2 25 to 30 mm Hg 2. Pao2 60 to 100 mm Hg, Paco2 30 to 35 mm Hg 3. Pao2 80 to 100 mm Hg, Paco2 25 to 30 mm Hg 4. Pao2 80 to 100 mm Hg, Paco2 35 to 40 mm Hg

3. Pao2 80 to 100 mm Hg, Paco2 25 to 30 mm Hg Rationale: Hyperventilation with a Paco2 of 25 to 30 mm Hg causes cerebral vasoconstriction, which decreases intracranial blood volume and ICP. The Pao2 is not allowed to fall below 80 mm Hg, to prevent cerebral vasodilation from hypoxemia. Therefore, the remaining options are incorrect.

A client with a neurological problem is experiencing hyperthermia. Which measure would be least appropriate for the nurse to use in trying to lower the client's body temperature? 1. Giving tepid sponge baths 2. Applying a hypothermia blanket 3. Placing ice packs in the axilla and groin areas 4. Administering acetaminophen (Tylenol) per protocol

3. Placing ice packs in the axilla and groin areas Rationale: Standard measures to lower body temperature include removing bed covers, providing cool sponge baths, using an electric fan in the room, administering acetaminophen, and placing a hypothermia blanket under the client. Ice packs are not used because they could cause shivering, which increases cellular oxygen demands, with the potential for increased intracranial pressure. Source: IGGY

The nurse is preparing for the admission of a client with a suspected diagnosis of herpes simplex encephalitis. The nurse anticipates that which diagnostic test will be prescribed to confirm this diagnosis? 1. Lumbar puncture 2. Electroencephalogram (EEG) 3. Polymerase chain reaction (PCR) 4. Computed tomography (CT) scan

3. Polymerase chain reaction (PCR) Rationale: The diagnosis of herpes simplex encephalitis can be made by a polymerase chain reaction (PCR) test (usually through a blood specimen) to detect viral DNA or ribonucleic acid (RNA) in the cerebrospinal fluid (CSF). Source: IGGY

The nurse is preparing to care for a client after a lumbar puncture. The nurse should plan to place the client in which best position immediately after the procedure? 1. Prone in semi-Fowler's position 2. Supine in semi-Fowler's position 3. Prone with a small pillow under the abdomen 4. Lateral with the head slightly lower than the rest of the body

3. Prone with a small pillow under the abdomen Rationale: After the procedure, the client assumes a flat position. If the client is able, a prone position with a pillow under the abdomen is the best position. This position helps reduce cerebrospinal fluid leakage and decreases the likelihood of post-lumbar puncture headache. Source: Pagana, Pagana

A nurse is reading a computer printout of the results of a cerebrospinal fluid (CSF) analysis performed on an adult client who has undergone lumbar puncture. The nurse determines that which is an abnormal finding? 1. Red blood cells, 0 2. Glucose, 52 mg/dL 3. Protein, 100 mg/dL 4. White blood cells, 3 cells/mm3

3. Protein, 100 mg/dL Rationale: Protein (15 to 45 mg/dL) and glucose (50 to 75 mg/dL) normally are present in CSF; however, the protein level for this client is above the expected range. The adult with normal CSF has no red blood cells in the CSF. The client may have small numbers of white blood cells (0 to 5 cells/mm3)

A client with Guillain-Barré syndrome has ascending paralysis and is intubated and receiving mechanical ventilation. Which strategy should the nurse incorporate in the plan of care to help the client cope with this illness? 1. Giving client full control over care decisions and restricting visitors 2. Providing positive feedback and encouraging active range of motion 3. Providing information, giving positive feedback, and encouraging relaxation 4. Providing intravenously administered sedatives, reducing distractions, and limiting visitors

3. Providing information, giving positive feedback, and encouraging relaxation Rationale: The client with Guillain-Barré syndrome experiences fear and anxiety from the ascending paralysis and sudden onset of the disorder. The nurse can alleviate these fears by providing accurate information about the client's condition, giving expert care and positive feedback to the client, and encouraging relaxation and distraction. The family can become involved with selected care activities and provide diversion for the client as well. Source: IGGY

The nurse is reviewing the medical records of a client admitted to the nursing unit with a diagnosis of a thrombotic brain attack (stroke). The nurse would expect to note that which is documented in the assessment data section of the record? 1. Sudden loss of consciousness occurred. 2. Signs and symptoms occurred suddenly. 3. The client experienced paresthesias a few days before admission to the hospital. 4. The client complained of a severe headache, which was followed by sudden onset of paralysis.

3. The client experienced paresthesias a few days before admission to the hospital. Rationale: Cerebral thrombosis does not occur suddenly. In the few hours or days preceding a thrombotic brain attack (stroke), the client may experience a transient loss of speech, hemiplegia, or paresthesias on one side of the body. Signs and symptoms of thrombotic brain attack (stroke) vary but may include dizziness, cognitive changes, or seizures. Headache is rare, but some clients with brain attack (stroke) experience signs and symptoms similar to those of cerebral embolism or intracranial hemorrhage.

The nurse is assessing a client for meningeal irritation and elicits a positive Brudzinski's sign. Which finding did the nurse observe? 1. The client rigidly extends the arms with pronated forearms and plantar flexion of the feet. 2. The client flexes a leg at the hip and knee and reports pain in the vertebral column when the leg is extended. 3. The client passively flexes the hip and knee in response to neck flexion and reports pain in the vertebral column. 4. The client's upper arms are flexed and held tightly to the sides of the body and the legs are extended and internally rotated.

3. The client passively flexes the hip and knee in response to neck flexion and reports pain in the vertebral column. Rationale: Brudzinski's sign is tested with the client in the supine position. The nurse flexes the client's head (gently moves the head to the chest) and there should be no reports of pain or resistance to the neck flexion. A positive Brudzinski's sign is observed if the client passively flexes the hip and knee in response to neck flexion and reports pain in the vertebral column. Kernig's sign also tests for meningeal irritation and is positive when the client flexes the legs at the hip and knee and complains of pain along the vertebral column when the leg is extended.

The home health nurse visits a client who is taking phenytoin (Dilantin) for control of seizures. During the assessment, the nurse notes that the client is taking birth control pills. Which information should the nurse include in the teaching plan? 1. Pregnancy should be avoided while taking phenytoin. 2. The client may stop the medication if it is causing severe gastrointestinal effects. 3. There is the potential of decreased effectiveness of birth control pills while taking phenytoin. 4. There is the increased risk of thrombophlebitis while taking phenytoin and birth control pills together.

3. There is the potential of decreased effectiveness of birth control pills while taking phenytoin. Rationale: Phenytoin enhances the rate of estrogen metabolism, which can decrease the effectiveness of some birth control pills. Source: Kee Hayes

A decision has just been made to give tissue plasminogen activator (t-PA) (Activase) to a client. The nurse should obtain which supply for standard use as part of safe nursing care related to this medication? 1. Flashlight 2. Pulse oximeter 3. Suction equipment 4. Occult blood test strips

4. Occult blood test strips Rationale: Activase is a thrombolytic medication that dissolves thrombi or emboli. Bleeding is a frequent and potentially severe adverse effect of therapy. The nurse assesses for signs of bleeding in clients receiving this therapy using occult blood test strips to test urine, stool, or nasogastric drainage

The nurse is performing an assessment on a client with a diagnosis of thrombotic brain attack (stroke). Which assessment question would elicit data specific to this type of stroke? 1. "Have you had any headaches in the past few days?" 2. "Have you recently been having difficulty with seeing at nighttime?" 3. "Have you had any sudden episodes of passing out in the past few days?" 4. "Have you had any numbness or tingling or paralysis-type feelings in any of your extremities recently?"

4. "Have you had any numbness or tingling or paralysis-type feelings in any of your extremities recently?" Rationale: Cerebral thrombosis (thrombotic stroke) does not occur suddenly. In the few days or hours preceding the thrombotic stroke, the client may experience a transient loss of speech, hemiparesis, or paresthesias on one side of the body. Signs and symptoms of this type of stroke vary but may also include dizziness, cognitive changes, or seizures. Headache is rare, but some clients with stroke experience signs and symptoms similar to those of cerebral embolism or intracranial hemorrhage. The client does not complain of difficulty with night vision as part of this clinical problem. In addition, most clients do not have repeated episodes of loss of consciousness.

The home care nurse is preparing to visit a client with a diagnosis of trigeminal neuralgia (tic douloureux). When performing the assessment, the nurse should plan to ask the client which question to elicit the most specific information regarding this disorder? 1. "Do you have any visual problems?" 2. "Are you having any problems hearing?" 3. "Do you have any tingling in the face region?" 4. "Is the pain experienced a stabbing type of pain?"

4. "Is the pain experienced a stabbing type of pain?" Rationale: Trigeminal neuralgia is characterized by spasms of pain that start suddenly and last for seconds to minutes. The pain often is characterized as stabbing or as similar to an electric shock. It is accompanied by spasms of facial muscles that cause twitching of parts of the face or mouth, or closure of the eye. Source: IGGY

The nurse is admitting a client to the short-stay unit after a myelogram. A water-based contrast agent was used. Which activity restrictions should the nurse should plan for the client? 1. Bed rest for 2 to 4 hours, with the head of bed flat 2. Bed rest for 6 to 8 hours, with the head of bed flat 3. Bed rest for 2 to 4 hours, with the head of bed elevated 30 degrees 4. Bed rest for 6 to 8 hours, with the head of bed elevated 30 degrees

4. Bed rest for 6 to 8 hours, with the head of bed elevated 30 degrees Rationale: After a myelogram, the client is placed on bed rest for 6 to 8 hours. If a water-based contrast medium was used, the client is positioned with the head of bed elevated 30 degrees. If an oil-based contrast study was done, the head of the bed is positioned flat. Source: Pagana, Pagana

A client is scheduled to begin therapy with carbamazepine (Tegretol). The nurse should assess the results of which test(s) before administering the first dose of this medication to the client? 1. Liver function tests 2. Renal function tests 3. Pancreatic enzyme studies 4. Complete blood cell count

4. Complete blood cell count Rationale: Carbamazepine (Tegretol) can cause leukopenia, anemia, thrombocytopenia, and, very rarely, fatal aplastic anemia. To reduce the risk of serious hematological effects, a complete blood cell count should be done before treatment and periodically thereafter. This medication should be avoided in clients with preexisting hematological abnormalities. The client also is told to report the occurrence of fever, sore throat, pallor, weakness, infection, easy bruising, and petechiae.

The nurse is admitting a client with Guillain-Barré syndrome to the nursing unit. The client has ascending paralysis to the level of the waist. Knowing the complications of the disorder, the nurse should bring which most essential items into the client's room? 1. Nebulizer and pulse oximeter 2. Blood pressure cuff and flashlight 3. Flashlight and incentive spirometer 4. Electrocardiographic monitoring electrodes and intubation tray

4. Electrocardiographic monitoring electrodes and intubation tray Rationale: The client with Guillain-Barré syndrome is at risk for respiratory failure because of ascending paralysis. An intubation tray should be available for use. Another complication of this syndrome is cardiac dysrhythmias, which necessitates the use of electrocardiographic monitoring. Because the client is immobilized, the nurse should assess for deep vein thrombosis and pulmonary embolism routinely. Although items in the incorrect options may be used in care, they are not the most essential items from the options provided.

The nurse is caring for a client in the emergency department who has sustained a head injury. The client momentarily lost consciousness at the time of the injury and then regained it. The client now has lost consciousness again. The nurse takes quick action, knowing that this sequence is compatible with which most likely condition? 1. Concussion 2. Skull fracture 3. Subdural hematoma 4. Epidural hematoma

4. Epidural hematoma Rationale: The changes in neurological signs from an epidural hematoma begin with loss of consciousness as arterial blood collects in the epidural space and exerts pressure. The client regains consciousness as the cerebrospinal fluid is reabsorbed rapidly to compensate for the rising intracranial pressure. As the compensatory mechanisms fail, even small amounts of additional blood cause the intracranial pressure to rise rapidly, and the client's neurological status deteriorates quickly. Source: IGGY

The nurse has taught a client with a herniated lumbar disk about proper body mechanics and other items pertinent to low back care. The nurse determines that further instruction is needed if the client states the need to take which action? 1. Bend at the knees to pick up objects. 2. Increase fiber and fluid intake in the diet. 3. Strengthen the back muscles by swimming or walking. 4. Get out of bed by sitting straight up and swinging the legs over the side of the bed.

4. Get out of bed by sitting straight up and swinging the legs over the side of the bed. Rationale: The client is taught to get out of bed by sliding near the edge of the mattress. The client then rolls onto one side and pushes up from the bed using one or both arms. The client keeps the back straight and swings the legs over the side. Proper body mechanics includes bending at the knees, not the waist, to lift objects. Increasing fluid intake and dietary fiber helps prevent straining at stool, thereby preventing increases in intraspinal pressure. Walking and swimming are excellent exercises for strengthening lower back muscles.

The nurse is developing a plan of care for a client with a stroke (brain attack) who has right homonymous hemianopsia. Which should the nurse include in the plan of care for the client? 1. Place an eye patch on the left eye. 2. Place personal articles on the client's right side. 3. Approach the client from the right field of vision. 4. Instruct the client to turn the head to scan the right visual field.

4. Instruct the client to turn the head to scan the right visual field. Rationale: Homonymous hemianopsia is a loss of half of the visual field. The nurse instructs the client to scan the environment and stands within the client's intact field of vision. The nurse should not patch the eye because the client does not have double vision. The client should have objects placed in the intact fields of vision, and the nurse should approach the client from the intact side

The nurse is caring for a client with a diagnosis of right (nondominant) hemispheric stroke. The nurse notes that the client is alert and oriented to time and place. On the basis of these assessment findings, the nurse should make which interpretation? 1. Had a very mild stroke 2. Most likely suffered a transient ischemic attack 3. May have difficulty with language abilities only 4. Is likely to have perceptual and spatial disabilities

4. Is likely to have perceptual and spatial disabilities Rationale: The client with a right (nondominant) hemispheric stroke may be alert and oriented to time and place. These signs of apparent wellness often suggest that the client is less disabled than is the case. However, impulsivity and confusion in carrying out activities may be very real problems for these clients as a result of perceptual and spatial disabilities. The right hemisphere is considered specialized in sensory-perceptual and visual-spatial processing and awareness of body space. The left hemisphere is dominant for language abilities. Source: IGGY

The nurse in the neurological unit is caring for a client with a supratentorial lesion. The nurse assesses which measurement as the most critical index of central nervous system (CNS) dysfunction? 1. Temperature 2. Blood pressure 3. Ability to speak 4. Level of consciousness

4. Level of consciousness Rationale: Level of consciousness is the most critical index of CNS dysfunction. Changes in level of consciousness can indicate clinical improvement or deterioration. Although blood pressure, temperature, and ability to speak may be components of the assessment, the client's level of consciousness is the most critical index of CNS dysfunction. Source: IGGY

A client with a spinal cord injury is prone to experiencing autonomic dysreflexia. The nurse should avoid which measure to minimize the risk of occurrence? 1. Strict adherence to a bowel retraining program 2. Keeping the linen wrinkle-free under the client 3. Preventing unnecessary pressure on the lower limbs 4. Limiting bladder catheterization to once every 12 hours

4. Limiting bladder catheterization to once every 12 hours Rationale: The most frequent cause of autonomic dysreflexia is a distended bladder. Straight catheterization should be done every 4 to 6 hours (catheterization every 12 hours is too infrequent), and Foley catheters should be checked frequently to prevent kinks in the tubing. Constipation and fecal impaction are other causes, so maintaining bowel regularity is important. Other causes include stimulation of the skin from tactile, thermal, or painful stimuli. The nurse administers care to minimize risk in these areas.

A client with a spinal cord injury expresses little interest in food and is very particular about the choice of meals that are actually eaten. How should the nurse interpret this information? 1. Anorexia is a sign of clinical depression, and a referral to a psychologist is needed. 2. The client has compulsive habits that should be ignored so long as they are not harmful. 3. The client probably has a naturally slow metabolism, and the decreased nutritional intake will not matter. 4. Meal choices represent an area of client control and should be encouraged as much as is nutritionally reasonable.

4. Meal choices represent an area of client control and should be encouraged as much as is nutritionally reasonable. Rationale: Depression frequently may be seen in the client with spinal cord injury and may be exhibited as a loss of appetite. However, the client should be allowed to choose the types of food eaten and when they are eaten as much as is feasible because it is one of the few areas of control that the client has left.

The client is admitted to the hospital with a diagnosis of Guillain-Barré syndrome. Which past medical history finding makes the client most at risk for this disease? 1. Meningitis or encephalitis during the last 5 years 2. Seizures or trauma to the brain within the last year 3. Back injury or trauma to the spinal cord during the last 2 years 4. Respiratory or gastrointestinal infection during the previous month

4. Respiratory or gastrointestinal infection during the previous month Rationale: Guillain-Barré syndrome is a clinical syndrome of unknown origin that involves cranial and peripheral nerves. Many clients report a history of respiratory or gastrointestinal infection in the 1 to 4 weeks before the onset of neurological deficits. On occasion, the syndrome can be triggered by vaccination or surgery.

A client is experiencing delirium. The nurse concludes that which areas of the nervous system are affected? 1. Temporal lobe and frontal lobe 2. Hippocampus and frontal lobe 3. Limbic system and cerebral hemispheres 4. Reticular activating system and cerebral hemispheres

4. Reticular activating system and cerebral hemispheres Rationale: Insomnia, agitation, mania, and delirium are caused by excessive arousal of the reticular activating system in conjunction with the cerebral hemispheres. The temporal lobe, hippocampus, and frontal lobe are responsible for memory. The limbic system is responsible for feelings and affect. Source: IGGY

Ticlopidine (Ticlid) is prescribed for a client. The nurse plans to take which action before implementing this medication therapy? 1. Taking the client's blood pressure 2. Obtaining a prothrombin time (PT) 3. Taking the client's apical heart rate 4. Reviewing the results of the complete blood cell (CBC) count

4. Reviewing the results of the complete blood cell (CBC) count Rationale: Ticlopidine (Ticlid) is an antiplatelet agent that is used for the prevention of thrombotic stroke. Ticlopidine's effects last for the life of the platelet, 7 to 10 days. Ticlopidine also can cause neutropenia, which is an abnormally small number of mature white blood cells (WBCs). Baseline data from a CBC are necessary before implementation of therapy, and the nurse should monitor for neutropenia during this medication therapy. If this adverse effect does occur, therapy should be stopped. The effects of neutropenia are reversible within 1 to 3 weeks

A client is about to undergo a lumbar puncture. The nurse describes to the client that which position will be used during the procedure? 1. Side-lying with a pillow under the hip 2. Prone with a pillow under the abdomen 3. Prone in slight Trendelenburg's position 4. Side-lying with the legs pulled up and the head bent down onto the chest

4. Side-lying with the legs pulled up and the head bent down onto the chest Rationale: A client undergoing lumbar puncture is positioned lying on the side, with the legs pulled up to the abdomen and the head bent down onto the chest. This position helps open the spaces between the vertebrae and allows for easier needle insertion by the health care provider. The nurse remains with the client during the procedure to help the client maintain this position.

A client with trigeminal neuralgia asks the nurse what causes the painful episodes associated with the condition. The nurse's response is based on an understanding that the symptoms can be triggered by which process? 1. A local reaction to nasal stuffiness 2. A hypoglycemic effect on the cranial nerve 3. Release of catecholamines with infection or stress 4. Stimulation of the affected nerve by pressure and temperature

4. Stimulation of the affected nerve by pressure and temperature Rationale: The paroxysms of pain that accompany this neuralgia are triggered by stimulation of the terminal branches of the trigeminal nerve. Symptoms can be triggered by pressure from washing the face, brushing the teeth, shaving, eating, or drinking. Symptoms also can be triggered by thermal stimuli, such as a draft of cold air. Source: IGGY

A health care provider (HCP) is planning to perform a lumbar puncture on a client. The nurse knows that this procedure will allow access to which anatomical area for diagnostic testing? 1. Vertebrae 2. Spinal cord 3. Epidural space 4. Subarachnoid space

4. Subarachnoid space Rationale: The HCP performing a lumbar puncture is gaining access to the client's subarachnoid space. This area is often accessed for diagnostic testing procedures such as lumbar puncture or myelogram. The spaces between the vertebrae are widened when the client is positioned for lumbar puncture. The spinal needle passes between vertebrae, not into them. The spinal cord is not touched directly during lumbar puncture. The epidural space may be used to deliver medications to the client. Source: IGGY & Pagana, Pagana

The nurse is teaching a client with myasthenia gravis about the prevention of myasthenic and cholinergic crises. Which client activity suggests that teaching is most effective? 1. Eating large, well-balanced meals 2. Doing muscle-strengthening exercises 3. Doing all chores early in the day while less fatigued 4. Taking medications on time to maintain therapeutic blood levels

4. Taking medications on time to maintain therapeutic blood levels Rationale: Clients with myasthenia gravis are taught to space out activities over the day to conserve energy and restore muscle strength. Taking medications correctly to maintain blood levels that are not too low or too high is important. Muscle-strengthening exercises are not helpful and can fatigue the client. Overeating is a cause of exacerbation of symptoms, as is exposure to heat, crowds, erratic sleep habits, and emotional stress. Source: IGGY

The nurse is preparing a plan of care for a client with a diagnosis of brain attack (stroke). On reviewing the client's record, the nurse notes an assessment finding of anosognosia. The nursing care plan should address which manifestation related to this finding? 1. The client will be easily fatigued. 2. The client will have difficulty speaking. 3. The client will have difficulty swallowing. 4. The client will exhibit neglect of the affected side.

4. The client will exhibit neglect of the affected side. Rationale: In anosognosia, the client neglects the affected side of the body. The client either may ignore the presence of the affected side (often creating a safety hazard as a result of potential injuries) or may state that the involved arm or leg belongs to someone else. Options 1, 2, and 3 are not associated with anosognosia.

A client with a probable minor head injury resulting from a motor vehicle crash is admitted to the hospital for observation. The nurse leaves the cervical collar applied to the client in place until when? 1. The family comes to visit. 2. The nurse needs to do physical care. 3. The health care provider makes rounds. 4. The results of spinal radiography are known.

4. The results of spinal radiography are known. Rationale: There is a significant association between cervical spine injury and head injury. For this reason, the nurse leaves any form of spinal immobilization in place until spinal radiographs rule out fracture or other damage Source: IGGY

An older man is brought to the hospital emergency department by a neighbor who heard him talking and found him wandering in the street at 3 am. The nurse should first determine which data about the client? 1. His insurance status 2. Blood toxicology levels 3. Whether he ate his evening meal 4. Whether this is a change in his usual level of orientation

4. Whether this is a change in his usual level of orientation Rationale: The nurse should first determine whether this behavior represents a change in the client's neurological status. The next item to determine is when the client last ate. Blood toxicology levels may or may not be needed, but the health care provider would likely prescribe these. Insurance information must be obtained at some point but is not the priority from a clinical care viewpoint. Source: IGGY

The client diagnosed with atrial fibrillation has experienced a transient ischemic attack (TIA). Which medication would the nurse anticipate being ordered for the client on discharge? A. An oral anticoagulant medication. B. A beta-blocker medication. C. An anti-hyperuricemic medication. D. A thrombolytic medication.

A. An oral anticoagulant medication. Explanation: Thrombi form secondary to atrial fibrillation, therefore, an anticoagulant would be anticipated to prevent thrombi formation; and oral (warfarin [Coumadin]) at discharge verses intravenous. Beta blockers slow the heart rate and lower the blood pressure. Anti-hyperuricemic medication is given to clients with gout. Thrombolytic medication might have been given at initial presentation but would not be a drug prescribed at discharge.

The nurse is assessing a 37-year-old client diagnosed with multiple sclerosis. Which of the following symptoms would the nurse expect to find? A. Vision changes B. Absent deep tendon reflexes C. Tremors at rest D. Flaccid muscles

A. Vision changes Explanation: Vision changes, such as diplopia, nystagmus, and blurred vision, are symptoms of multiple sclerosis. Deep tendon reflexes may be increased or hyperactive — not absent. Babinski's sign may be positive. Tremors at rest aren't characteristic of multiple sclerosis; however, intentional tremors, or those occurring with purposeful voluntary movement, are common in clients with multiple sclerosis. Affected muscles are spastic, rather than flaccid.

A male client is admitted with a cervical spine injury sustained during a diving accident. When planning this client's care, the nurse should assign highest priority to which nursing diagnosis? A. Impaired physical mobility B. Ineffective breathing pattern C. Disturbed sensory perception (tactile) D. Self-care deficit: Dressing/grooming

B. Ineffective breathing pattern Explanation: Because a cervical spine injury can cause respiratory distress, the nurse should take immediate action to maintain a patent airway and provide adequate oxygenation. The other options may be appropriate for a client with a spinal cord injury — particularly during the course of recovery — but don't take precedence over a diagnosis of Ineffective breathing pattern.

A client admitted to the hospital with a subarachnoid hemorrhage has complaints of severe headache, nuchal rigidity, and projectile vomiting. The nurse knows lumbar puncture (LP) would be contraindicated in this client in which of the following circumstances? A. Vomiting continues B. Intracranial pressure (ICP) is increased C. The client needs mechanical ventilation D. Blood is anticipated in the cerebrospinal fluid (CSF)

B. Intracranial pressure (ICP) is increased Explanation: Sudden removal of CSF results in pressures lower in the lumbar area than the brain and favors herniation of the brain; therefore, LP is contraindicated with increased ICP. Vomiting may be caused by reasons other than increased ICP; therefore, LP isn't strictly contraindicated. An LP may be performed on clients needing mechanical ventilation. Blood in the CSF is diagnostic for subarachnoid hemorrhage and was obtained before signs and symptoms of ICP.

You are preparing to admit a patient with a seizure disorder. Which of the following actions can you delegate to LPN/LVN? A. Complete admission assessment. B. Set up oxygen and suction equipment. C. Place a padded tongue blade at bedside. D. Pad the side rails before patient arrives.

B. Set up oxygen and suction equipment. Explanation: The LPN/LVN can set up the equipment for oxygen and suctioning. The RN should perform the complete initial assessment. Padded side rails are controversial in terms of whether they actually provide safety and ay embarrass the patient and family. Tongue blades should not be at the bedside and should never be inserted into the patient's mouth after a seizure begins.

The nurse is teaching a female client with multiple sclerosis. When teaching the client how to reduce fatigue, the nurse should tell the client to: A. take a hot bath. B. rest in an air-conditioned room. C. increase the dose of muscle relaxants. D. avoid naps during the day.

B. rest in an air-conditioned room. Explanation: Fatigue is a common symptom in clients with multiple sclerosis. Lowering the body temperature by resting in an air-conditioned room may relieve fatigue; however, extreme cold should be avoided. A hot bath or shower can increase body temperature, producing fatigue. Muscle relaxants, prescribed to reduce spasticity, can cause drowsiness and fatigue. Planning for frequent rest periods and naps can relieve fatigue. Other measures to reduce fatigue in the client with multiple sclerosis include treating depression, using occupational therapy to learn energy conservation techniques, and reducing spasticity.

During the first 24 hours after thrombolytic therapy for ischemic stroke, the primary goal is to control the client's: A. Pulse B. Respirations C. Blood pressure D. Temperature

C. Blood pressure Explanation: Controlling the blood pressure is critical because an intracerebral hemorrhage is the major adverse effect of thrombolytic therapy. Blood pressure should be maintained according to physician and is specific to the client's ischemic tissue needs and risks of bleeding from treatment. Other vital signs are monitored, but the priority is blood pressure.

What is the expected outcome of thrombolytic drug therapy? A. Increased vascular permeability. B. Vasoconstriction. C. Dissolved emboli. D. Prevention of hemorrhage

C. Dissolved emboli. Explanation: Thrombolytic therapy is use to dissolve emboli and reestablish cerebral perfusion.

A client comes into the ER after hitting his head in an MVA. He's alert and oriented. Which of the following nursing interventions should be done first? A. Assess full ROM to determine extent of injuries B. Call for an immediate chest x-ray C. Immobilize the client's head and neck D. Open the airway with the head-tilt chin-lift maneuver

C. Immobilize the client's head and neck Explanation: All clients with a head injury are treated as if a cervical spine injury is present until x-rays confirm their absence. ROM would be contraindicated at this time. There is no indication that the client needs a chest x-ray. The airway doesn't need to be opened since the client appears alert and not in respiratory distress. In addition, the head-tilt chin-lift maneuver wouldn't be used until the cervical spine injury is ruled out.

A physician diagnoses a client with myasthenia gravis, prescribing pyridostigmine (Mestinon), 60 mg P.O. every 3 hours. Before administering this anticholinesterase agent, the nurse reviews the client's history. Which preexisting condition would contraindicate the use of pyridostigmine? A. Ulcerative colitis B. Blood dyscrasia C. Intestinal obstruction D. Spinal cord injury

C. Intestinal obstruction Explanation: Anticholinesterase agents such as pyridostigmine are contraindicated in a client with a mechanical obstruction of the intestines or urinary tract, peritonitis, or hypersensitivity to anticholinesterase agents. Ulcerative colitis, blood dyscrasia, and spinal cord injury don't contraindicate use of the drug.

Nurse Mary witnesses a neighbor's husband sustain a fall from the roof of his house. The nurse rushes to the victim and determines the need to opens the airway in this victim by using which method? A. Flexed position B. Head tilt-chin lift C. Jaw thrust maneuver D. Modified head tilt-chin lift

C. Jaw thrust maneuver Explanation: If a neck injury is suspected, the jaw thrust maneuver is used to open the airway. The head tilt-chin lift maneuver produces hyperextension of the neck and could cause complications if a neck injury is present. A flexed position is an inappropriate position for opening the airway.

A client is admitted with a spinal cord injury at the level of T12. He has limited movement of his upper extremities. Which of the following medications would be used to control edema of the spinal cord? A. Acetazolamide (Diamox) B. Furosemide (Lasix) C. Methylprednisolone (Solu-Medrol) D. Sodium bicarbonate

C. Methylprednisolone (Solu-Medrol) Explanation: High doses of Solu-Medrol are used within 24 hours of spinal injury to reduce cord swelling and limit neurological deficit. The other drugs aren't indicated in this circumstance.

A female client who was trapped inside a car for hours after a head-on collision is rushed to the emergency department with multiple injuries. During the neurologic examination, the client responds to painful stimuli with decerebrate posturing. This finding indicates damage to which part of the brain? A. Diencephalon B. Medulla C. Midbrain D. Cortex

C. Midbrain Explanation: Decerebrate posturing, characterized by abnormal extension in response to painful stimuli, indicates damage to the midbrain. With damage to the diencephalon or cortex, abnormal flexion (decorticate posturing) occurs when a painful stimulus is applied. Damage to the medulla results in flaccidity.

While in the ER, a client with C8 tetraplegia develops a blood pressure of 80/40, pulse 48, and RR of 18. The nurse suspects which of the following conditions? A. Autonomic dysreflexia B. Hemorrhagic shock C. Neurogenic shock D. Pulmonary embolism

C. Neurogenic shock Explanation: Symptoms of neurogenic shock include hypotension, bradycardia, and warm, dry skin due to the loss of adrenergic stimulation below the level of the lesion. Hypertension, bradycardia, flushing, and sweating of the skin are seen with autonomic dysreflexia. Hemorrhagic shock presents with anxiety, tachycardia, and hypotension; this wouldn't be suspected without an injury. Pulmonary embolism presents with chest pain, hypotension, hypoxemia, tachycardia, and hemoptysis; this may be a later complication of spinal cord injury due to immobility.

A 30-year-old was admitted to the progressive care unit with a C5 fracture from a motorcycle accident. Which of the following assessments would take priority? A. Bladder distension B. Neurological deficit C. Pulse ox readings D. The client's feelings about the injury

C. Pulse ox readings Explanation: After a spinal cord injury, ascending cord edema may cause a higher level of injury. The diaphragm is innervated at the level of C4, so assessment of adequate oxygenation and ventilation is necessary. Although the other options would be necessary at a later time, observation for respiratory failure is the priority.

Which of the following signs and symptoms of increased ICP after head trauma would appear first? A. Bradycardia B. Large amounts of very dilute urine C. Restlessness and confusion D. Widened pulse pressure

C. Restlessness and confusion Explanation: The earliest symptom of elevated ICP is a change in mental status. Bradycardia, widened pulse pressure, and bradypnea occur later. The client may void large amounts of very dilute urine if there's damage to the posterior pituitary.

A 78 year old client is admitted to the emergency department with numbness and weakness of the left arm and slurred speech. Which nursing intervention is priority? A. Prepare to administer recombinant tissue plasminogen activator (rt-PA). B. Discuss the precipitating factors that caused the symptoms. C. Schedule for A STAT computer tomography (CT) scan of the head. D. Notify the speech pathologist for an emergency consult.

C. Schedule for A STAT computer tomography (CT) scan of the head. Explanation: A CT scan will determine if the client is having a stroke or has a brain tumor or another neurological disorder. This would also determine if it is a hemorrhagic or ischemic accident and guide the treatment, because only an ischemic stroke can use rt-PA. This would make (1) not the priority since if a stroke was determined to be hemorrhagic, rt-PA is contraindicated. Discuss the precipitating factors for teaching would not be a priority and slurred speech would as indicate interference for teaching. Referring the client for speech therapy would be an intervention after the CVA emergency treatment is administered according to protocol.

A client with a subdural hematoma becomes restless and confused, with dilation of the ipsilateral pupil. The physician orders mannitol for which of the following reasons? A To reduce intraocular pressure B. To prevent acute tubular necrosis C. To promote osmotic diuresis to decrease ICP D. To draw water into the vascular system to increase blood pressure

C. To promote osmotic diuresis to decrease ICP Explanation: Mannitol promotes osmotic diuresis by increasing the pressure gradient, drawing fluid from intracellular to intravascular spaces. Although mannitol is used for all the reasons described, the reduction of ICP in this client is a concern.

Nurse Amber is caring for a client who underwent a lumbar laminectomy two days ago. Which of the following findings should the nurse consider abnormal? A. More back pain than the first postoperative day B. Paresthesia in the dermatomes near the wounds C. Urine retention or incontinence D. Temperature of 99.2° F (37.3° C)

C. Urine retention or incontinence Explanation: Urine retention or incontinence may indicate cauda equina syndrome, which requires immediate surgery. An increase in pain on the second postoperative day is common because the long-acting local anesthetic, which may have been injected during surgery, will wear off. While paresthesia is common after surgery, progressive weakness or paralysis may indicate spinal nerve compression. A mild fever is also common after surgery but is considered significant only if it reaches 101° F (38.3° C).

A female client admitted to an acute care facility after a car accident develops signs and symptoms of increased intracranial pressure (ICP). The client is intubated and placed on mechanical ventilation to help reduce ICP. To prevent a further rise in ICP caused by suctioning, the nurse anticipates administering which drug endotracheally before suctioning? A. phenytoin (Dilantin) B. mannitol (Osmitrol) C. lidocaine (Xylocaine) D. furosemide (Lasix)

C. lidocaine (Xylocaine) Explanation: Administering lidocaine via an endotracheal tube may minimize elevations in ICP caused by suctioning. Although mannitol and furosemide may be given to reduce ICP, they're administered parenterally, not endotracheally. Phenytoin doesn't reduce ICP directly but may be used to abolish seizures, which can increase ICP. However, phenytoin isn't administered endotracheally.

For a male client with suspected increased intracranial pressure (ICP), a most appropriate respiratory goal is to: A. prevent respiratory alkalosis. B. lower arterial pH. C. promote carbon dioxide elimination. D. maintain partial pressure of arterial oxygen (PaO2) above 80 mm Hg

C. promote carbon dioxide elimination. Explanation: The goal of treatment is to prevent acidemia by eliminating carbon dioxide. That is because an acid environment in the brain causes cerebral vessels to dilate and therefore increases ICP. Preventing respiratory alkalosis and lowering arterial pH may bring about acidosis, an undesirable condition in this case. It isn't necessary to maintain a PaO2 as high as 80 mm Hg; 60 mm Hg will adequately oxygenate most clients

During recovery from a cerebrovascular accident (CVA), a female client is given nothing by mouth, to help prevent aspiration. To determine when the client is ready for a liquid diet, the nurse assesses the client's swallowing ability once each shift. This assessment evaluates: A. cranial nerves I and II. B. cranial nerves III and V. C. cranial nerves VI and VIII. D. cranial nerves IX and X.

D. cranial nerves IX and X. Explanation: Swallowing is a motor function of cranial nerves IX and X. Cranial nerves I, II, and VIII don't possess motor functions. The motor functions of cranial nerve III include extraocular eye movement, eyelid elevation, and pupil constriction. The motor function of cranial nerve V is chewing. Cranial nerve VI controls lateral eye movement.

The nurse is caring for a client who sustained a spinal cord injury. During administration of morning care, the client begins to exhibit signs and symptoms of autonomic dysreflexia. Which initial nursing action should the nurse take? 1. Elevate the head of the bed. 2. Examine the rectum digitally. 3. Assess the client's blood pressure. 4. Place the client in the prone position.

1. Elevate the head of the bed. Rationale: Autonomic dysreflexia is a serious complication that can occur in the spinal cord-injured client. Once the syndrome is identified, the nurse elevates the head of the client's bed and then examines the client for the source of noxious stimuli. The nurse also assesses the client's blood pressure, but the initial action would be to elevate the head of the bed. The client would not be placed in the prone position. Source: IGGY

A client with a diagnosis of trigeminal neuralgia is started on a regimen of carbamazepine (Tegretol). The nurse provides instructions to the client about the side/adverse effects of the medication. Which client statement indicates an understanding of the side/adverse effects of the medication? 1. "I will report a fever or sore throat to my health care provider." 2. "I must brush my teeth frequently to avoid damage to my gums." 3. "If I notice ringing in my ears that doesn't stop, I'll seek medical attention." 4. "If I notice a pink color to my urine, I will stop the medication and call my health care provider (HCP)."

1. "I will report a fever or sore throat to my health care provider." Rationale: Agranulocytosis is adverse effect of carbamazepine and places the client at risk for infection. If a fever or a sore throat develops, the HCP should be notified. Gum damage, ringing in the ears, and pink-colored urine are not effects associated with this medication.

The nurse is caring for a client who is in the chronic phase of stroke (brain attack) and has a right-sided hemiparesis. The nurse identifies that the client is unable to feed self. Which is a priority nursing intervention? 1. Assist the client to eat with the left hand to build strength. 2. Provide a pureed diet that is easy for the client to swallow. 3. Inform the client that a feeding tube will be placed if progress is not made. 4. Provide a variety of foods on the meal tray to stimulate the client's appetite.

1. Assist the client to eat with the left hand to build strength. Rationale: Right-sided hemiparesis is weakness of the right arm and leg. The nurse should teach the client to use both sides of the body to increase strength and build endurance. Option 2 is incorrect. The question does not mention swallowing difficulty, so there is no need to puree the food. Option 3 is incorrect. That information would come from the health care provider. Option 4 is incorrect. The problem is not the food selection but the client's ability to eat the food independently. Source: IGGY

The home health nurse is visiting a client with myasthenia gravis and is discussing methods to minimize the risk of aspiration during meals related to decreased muscle strength. Which suggestions should the nurse give to the client? Select all that apply. 1. Chew food thoroughly. 2. Cut food into very small pieces. 3. Sit straight up in the chair while eating. 4. Lift the head while swallowing liquids. 5. Swallow when the chin is tipped slightly downward to the chest.

1. Chew food thoroughly. 2. Cut food into very small pieces. 3. Sit straight up in the chair while eating. 5. Swallow when the chin is tipped slightly downward to the chest. Source: IGGY

The nurse reviews the health care provider's (HCP) prescriptions for a client with Guillain-Barré syndrome. Which prescription written by the HCP should the nurse question? 1. Clear liquid diet 2. Bilateral calf measure 3. Monitor vital signs frequently 4. Passive range-of-motion (ROM) exercises

1. Clear liquid diet Rationale: Clients with Guillain-Barré syndrome have dysphagia. Clients with dysphagia are more likely to aspirate clear liquids than thick or semisolid foods. Passive ROM exercises can help prevent contractures, and assessing calf measurements can help detect deep vein thrombosis, for which these clients are at risk. Because clients with Guillain-Barré syndrome are at risk for hypotension or hypertension, bradycardia, and respiratory depression, frequent monitoring of vital signs is required. Source: IGGY

A client who has had a stroke (brain attack) has residual dysphagia. When a diet prescription is initiated, the nurse should avoid which action? 1. Giving the client thin liquids 2. Thickening liquids to the consistency of oatmeal 3 Placing food on the unaffected side of the mouth 4. Allowing plenty of time for chewing and swallowing

1. Giving the client thin liquids Rationale: The client with dysphagia is started on a diet only after the gag and swallow reflexes have returned. The client is assisted with meals as needed and is given ample time to chew and swallow. Food is placed on the unaffected side of the mouth. Liquids are thickened to avoid aspiration.

A client with multiple sclerosis tells a home health care nurse that she is having increasing difficulty in transferring from the bed to a chair. What is the initial nursing action? 1. Observe the client demonstrating the transfer technique. 2. Start a restorative nursing program before an injury occurs. 3. Seize the opportunity to discuss potential nursing home placement. 4. Determine the number of falls that the client has had in recent weeks.

1. Observe the client demonstrating the transfer technique. Rationale: Observation of the client's transfer technique is the initial intervention. Starting a restorative program is important but not unless an assessment has been completed first. Discussing nursing home placement would be inappropriate in view of the information provided in the question. Determining the number of falls is another important intervention, but observing the transfer technique should be done first. Source: IGGY

A client with myasthenia gravis who is taking neostigmine (Prostigmin) is experiencing frequent exacerbations of myasthenic crisis and cholinergic crisis. The nurse teaches the client that it is most important that this medication be taken in which manner? 1. On time 2. On an empty stomach 3. Double-dosed if one dose is missed 4. Titrated for dosage, depending on the symptoms

1. On time Rationale: The client should take neostigmine exactly on time. Taking the medication early or late could result in myasthenic or cholinergic crisis. Taking the medication on time is especially important for the client with dysphagia because the client may not be able to swallow the medication if it is given late. These clients are taught to set an alarm clock to remind them of dosage times. The medication should be administered with food or milk to minimize side effects and adverse effects. The client should never skip or double-up on missed doses or titrate the dose, depending on symptoms. The client needs to take the medication exactly as prescribed.

The nurse is preparing a plan of care for a client with a diagnosis of amyotrophic lateral sclerosis (ALS). On assessment, the nurse notes that the client is severely dysphagic. Which intervention should be included in the care plan for this client? Select all that apply. 1. Provide oral hygiene after each meal. 2. Assess swallowing ability frequently. 3. Allow the client sufficient time to eat. 4. Maintain a suction machine at the bedside. 5. Provide a full liquid diet for ease in swallowing.

1. Provide oral hygiene after each meal. 2. Assess swallowing ability frequently. 3. Allow the client sufficient time to eat. 4. Maintain a suction machine at the bedside. Rationale: A client who is severely dysphagic is at risk for aspiration. Swallowing is assessed frequently. The client should be given a sufficient amount of time to eat. Semisoft foods are easiest to swallow and require less chewing. Oral hygiene is necessary after each meal. Suctioning should be available for clients who experience dysphagia and are at risk for aspiration. Source: IGGY

The nurse is teaching a client with paraplegia measures to maintain skin integrity. Which instruction will be most helpful to the client? 1. Shift weight every 2 hours while in a wheelchair. 2. Change bed sheets every other week to maintain cleanliness. 3. Place a pillow on the seat of the wheelchair to provide extra comfort. 4. Use a mirror to inspect for redness and skin breakdown twice a week.

1. Shift weight every 2 hours while in a wheelchair. Rationale: To maintain skin integrity, the client should shift weight in the wheelchair every 2 hours and use a pressure relief pad. A pillow is not sufficient to relieve the pressure. While the client is in bed, the bottom sheet should be free of wrinkles and wetness. Sheets should be changed as needed and more frequently than every other week. The client should use a mirror to inspect the skin twice daily (morning and evening) to assess for redness, edema, and breakdown. General additional measures include a nutritious diet and meticulous skin care. Source: IGGY

A client who had a brain attack (stroke) has right-sided hemianopsia. What should the nurse plan to do to help the client adapt to this problem? 1. Teach the client to scan the environment. 2. Place all objects within the left visual field. 3. Place all objects within the right visual field. 4. Ensure that the family brings the client's eyeglasses to hospital.

1. Teach the client to scan the environment. Rationale: Hemianopsia is blindness in half the visual field. The client with hemianopsia is taught to scan the environment. This allows the client to take in the entirety of the visual field, which is necessary for proper functioning within the environment and helps to prevent injury to the client. Options 2 and 3 will not help the client adapt to this visual impairment. Eyeglasses are useful if the client already wears them, but they will not correct this visual-field deficit. Source: IGGY

A client is diagnosed with Bell's palsy. The nurse assessing the client expects to note which symptom? 1. A symmetrical smile 2. Difficulty closing the eyelid on the affected side 3. Narrowing of the palpebral fissure on the affected side 4. Paroxysms of excruciating pain in the lips and cheek on the affected side

2. Difficulty closing the eyelid on the affected side Rationale: The facial drooping associated with Bell's palsy makes it difficult for the client to close the eyelid on the affected side. A widening of the palpebral fissure (the opening between the eyelids) and an asymmetrical smile are seen with Bell's palsy. Paroxysms of excruciating pain are characteristic of trigeminal neuralgia.

The nurse is positioning a client who has increased intracranial pressure. Which position should the nurse avoid? 1. Head midline 2. Head turned to the side 3. Neck in neutral position 4. Head of bed elevated 30 to 45 degrees

2. Head turned to the side Rationale: The head of a client with increased intracranial pressure should be kept in a neutral midline position. The nurse should avoid flexing or extending the client's neck or turning the head from side to side. The head of the bed should be raised to 30 to 45 degrees. Use of proper positions promotes venous drainage from the cranium to keep intracranial pressure down

A client has a high level of carbon dioxide (CO2) in the bloodstream, as measured by arterial blood gases. A nurse reviewing the client's record plans care, knowing that a high CO2 level will have which effect on circulation to the brain? 1. It will cause arteriovenous shunting. 2. It will cause vasodilation of blood vessels in the brain. 3. It will cause blood vessels in the circle of Willis to collapse. 4. It will cause hyperresponsiveness of blood vessels in the brain.

2. It will cause vasodilation of blood vessels in the brain. Rationale: CO2 is one of the metabolic end products that can alter the tone of the blood vessels in the brain. High CO2 levels cause vasodilation, which may cause headache, whereas low CO2 levels cause vasoconstriction, which may cause lightheadedness. Source: IGGY

The nurse is evaluating the respiratory outcomes for a client with Guillain-Barré syndrome. The nurse determines that which is the least optimal outcome for the client? 1. Spontaneous breathing 2. Oxygen saturation of 98% 3. Adventitious breath sounds 4. Vital capacity within normal range

3. Adventitious breath sounds Rationale: Satisfactory respiratory outcomes for a client with Guillain-Barré syndrome include clear breath sounds on auscultation, spontaneous breathing, normal vital capacity, normal arterial blood gas levels, and normal pulse oximetry. Adventitious breath sounds are an abnormal finding. Source: IGGY

A client has suffered a head injury affecting the occipital lobe of the brain. The nurse anticipates that the client may experience difficulty with which sense? 1. Smell 2. Taste 3. Vision 4. Hearing

3. Vision Rationale: The occipital lobe is responsible for reception of vision and contains visual association areas. This area of the brain helps the individual to visually recognize and understand the surroundings. The other senses listed are not a function of the occipital lobe. Source: IGGY

The nurse is performing an assessment on a client suspected of having trigeminal neuralgia (tic douloureux). Which assessment question would elicit data specific to this disorder? 1. "Have you had any facial paralysis?" 2. "Have you noticed that your eyelid has been drooping?" 3. "Have you had any numbness and tingling in your face?" 4. "Have you had any sharp pain or any twitching in any part of your face?"

4. "Have you had any sharp pain or any twitching in any part of your face?" Rationale: Trigeminal neuralgia is characterized by spasms of pain that start suddenly and last from seconds to minutes. The pain often is described as either stabbing or similar to an electric shock. It is accompanied by spasms of the facial muscles that cause twitching of parts of the face or mouth, or closure of the eye. Source: IGGY

A client is suspected of having myasthenia gravis. Edrophonium (Enlon) is administered intravenously to determine the diagnosis. Which indicates that the client may have myasthenia gravis? 1. Joint pain following administration of the medication 2. Feelings of faintness, dizziness, hypotension, and signs of flushing in the client 3. A decrease in muscle strength within 30 to 60 seconds following administration of the medication 4. An increase in muscle strength within 30 to 60 seconds following administration of the medication

4. An increase in muscle strength within 30 to 60 seconds following administration of the medication Rationale: Edrophonium is a short-acting acetylcholinesterase inhibitor used as a diagnostic agent. When a client with suspected myasthenia gravis the health care provider will administer a edrophonium test. When a dose is administered intravenously, an increase in muscle strength should be seen in 30 to 60 seconds. If no response occurs, another dose of edrophonium is given over the next 2 minutes, and muscle strength is tested again. If no increase in muscle strength occurs with this higher dose, the muscle weakness is not caused by myasthenia gravis. Clients receiving injections of this medication commonly demonstrate a drop in blood pressure, feel faint and dizzy, and are flushed.

The nurse is caring for the client who suffered a spinal cord injury 48 hours ago. What should the nurse assess for when monitoring for gastrointestinal complications? 1. A history of diarrhea 2. A flattened abdomen 3. Hyperactive bowel sounds 4. Hematest-positive nasogastric tube drainage

4. Hematest-positive nasogastric tube drainage Rationale: Development of a stress ulcer also can occur after spinal cord injury and can be detected by Hematest-positive nasogastric tube aspirate or stool. The client is also at risk for paralytic ileus, which is characterized by the absence of bowel sounds and abdominal distention. A history of diarrhea is irrelevant. Source: IGGY

The nurse is caring for an unconscious client who is experiencing persistent hyperthermia with no signs of infection. The nurse interprets that the hyperthermia may be related to damage to the client's thermoregulatory center in which structure? 1. Cerebrum 2. Cerebellum 3. Hippocampus 4. Hypothalamus

4. Hypothalamus Rationale: Hypothalamic damage causes persistent hyperthermia, which also may be called central fever. It is characterized by a persistent high fever with no diurnal variation. Another characteristic feature is absence of sweating Source: IGGY

A nurse is providing instructions to a client beginning medication therapy with divalproex sodium (Depakote) for treatment of absence seizures. The nurse instructs the client that which represents the most frequent side effect of this medication? 1. Tinnitus 2. Irritability 3. Blue vision 4. Nausea and vomiting

4. Nausea and vomiting Rationale: The most frequent side effects of medication therapy with divalproex sodium (Depakote) are gastrointestinal (GI) disturbances, such as nausea, vomiting, and indigestion. The items in the other options are incorrect.

A nurse is caring for a client with an intracranial pressure (ICP) monitoring device. The nurse should become most concerned if the ICP readings drifted to and stayed in the vicinity of which finding? 1. 5 mm Hg 2. 8 mm Hg 3. 14 mm Hg 4. 22 mm Hg

4. 22 mm Hg Rationale: Normal ICP readings range from 5 to 15 mm Hg pressure. Pressures greater than 20 mm Hg are considered to represent increased ICP, which seriously impairs cerebral perfusion. Source: IGGY

A thymectomy accomplished via a median sternotomy approach is performed in a client with a diagnosis of myasthenia gravis. The nurse develops a postoperative plan of care for the client that should include which intervention? 1. Monitor the chest tube drainage. 2. Restrict visitors for 24 hours postoperatively. 3. Maintain intravenous infusion of lactated Ringer's solution. 4. Avoid administering pain medication to prevent respiratory depression.

1. Monitor the chest tube drainage. Rationale: A thymectomy may be used for management of clients with myasthenia gravis. The procedure is performed through a median sternotomy or a transcervical approach. Postoperatively the client will have a chest tube in the mediastinum. Lactated intravenous solutions usually are avoided because they can increase weakness. Pain medication is administered as needed, but the client is monitored closely for respiratory depression. There is no reason to restrict visitors. Source: IGGY

A client arrives in the hospital emergency department with a closed head injury to the right side of the head caused by an assault with a baseball bat. The nurse assesses the client neurologically, looking primarily for motor response deficits that involve which area? 1. The left side of the body 2. The right side of the body 3. Both sides of the body equally 4. Cranial nerves only, such as speech and pupillary response

1. The left side of the body Rationale: Motor responses such as weakness and decreased movement will be seen on the side of the body that is opposite an area of head injury. Contralateral deficits result from compression of the cortex of the brain or the pyramidal tracts. Depending on the severity of the injury, the client may have a variety of neurological deficits. Source: IGGY

A client has sustained damage to Wernicke's area in the temporal lobe from a stroke (brain attack). The nurse anticipates that the client will have difficulty with which function? 1. Articulating words 2. Understanding language 3. Moving one side of the body 4. Recalling events in the remote past

2. Understanding language Rationale: Wernicke's area consists of a small group of cells in the temporal lobe whose function is the understanding of language. Damage to Broca's area is responsible for aphasia. Source: IGGY

A nurse preparing to administer carbamazepine (Tegretol) notices each of the following items on the client's breakfast tray. Which item should be a cause for concern and should be removed from the tray? 1. Carton of milk 2. Scrambled eggs 3. Grapefruit juice 4. Toast with honey

3. Grapefruit juice Rationale: Grapefruit juice can increase peak and trough levels of carbamazepine by 40%. Accordingly, clients taking the medication should be advised to avoid grapefruit juice. The other foods can be taken with this medication.

A nurse caring for a client following craniotomy who has a supratentorial incision understands that the client should most likely be maintained in which position? 1. Prone position 2. Supine position 3. Semi-Fowler's position 4. Dorsal recumbent position

3. Semi-Fowler's position Rationale: In supratentorial surgery (surgery above the brain's tentorium), the client's head is usually elevated 30 degrees to promote venous outflow through the jugular veins. The client's head or the head of the bed is not lowered in the acute phase of care after supratentorial surgery. An exception to this position is the client who has undergone evacuation of a chronic subdural hematoma, but a health care provider's (HCP) prescription is required for positions other than those involving head elevation. Additionally, the health care provider's (HCP) prescription regarding positioning is always checked and agency procedures are always followed. Source: IGGY

A home care nurse visits a client at home. Clonazepam (Klonopin) has been prescribed for the client, and the nurse teaches the client about the medication. Which client statement indicates that further teaching is necessary? 1. "My drowsiness will decrease over time with continued treatment." 2. "I should take my medicine with food to avoid any stomach problems." 3. "I can take my medicine at bedtime if it tends to make me feel drowsy." 4. "If I experience slurred speech, this problem will disappear in about 8 weeks."

4. "If I experience slurred speech, this problem will disappear in about 8 weeks." Rationale: Clients who are experiencing signs and symptoms of toxicity with the administration of clonazepam exhibit slurred speech, sedation, confusion, respiratory depression, hypotension, and eventually coma. The medication may be taken with food to decrease gastrointestinal irritation. Some drowsiness may occur but will decrease with continued use. Options 1, 2, and 3 all are correct and represent an accurate understanding of the medication. Source: Kee Hayes

Carbamazepine (Tegretol) has been prescribed for a client. The nurse should tell the client that which blood test will be done periodically while the client is taking this medication? 1. Lipase level 2. Amylase level 3. Ammonia level 4. Complete blood cell (CBC) count

4. Complete blood cell (CBC) count Rationale: Carbamazepine is classified as an iminostilbene derivative and is used as an anticonvulsant, antineuralgic, antimanic, and antipsychotic. The medication can cause blood dyscrasias as an adverse effect, and the client should have a CBC done before therapy and periodically during therapy. Additional laboratory tests that should be done include a serum iron determination, urinalysis, blood urea nitrogen determination, and a carbamazepine level.

A client with myasthenia gravis arrives at the hospital emergency department in suspected crisis. The health care provider plans to administer edrophonium to differentiate between myasthenic and cholinergic crises. The nurse ensures that which medication is available in the event that the client is in cholinergic crisis? 1. Atropine sulfate 2. Morphine sulfate 3. Protamine sulfate 4. Pyridostigmine bromide

1. Atropine sulfate Rationale: Clients with cholinergic crisis have experienced overdosage of medication. Edrophonium will exacerbate symptoms in cholinergic crisis to the point at which the client may need intubation and mechanical ventilation. Intravenous atropine sulfate is used to reverse the effects of these anticholinesterase medications.Morphine sulfate and pyridostigmine bromide would worsen the symptoms of cholinergic crisis. Source: IGGY

The nurse is caring for a client after a craniotomy and monitors the client for signs of increased intracranial pressure (ICP). Which finding, if noted in the client, would indicate an early sign of increased ICP? 1. Confusion 2. Bradycardia 3. Sluggish pupils 4. A widened pulse pressure

1. Confusion Rationale: Early manifestations of increased ICP are subtle and often may be transient, lasting for only a few minutes in some cases. These early clinical manifestations include episodes of confusion, drowsiness, and slight pupillary and breathing changes. Later manifestations include a further decrease in the level of consciousness, a widened pulse pressure, and bradycardia. Cheyne-Stokes respiratory pattern, or a hyperventilation respiratory pattern; pupillary sluggishness and dilatation appear in the late stages. Source: IGGY

The nurse is caring for a client who is on bed rest as part of aneurysm precautions. The nurse should avoid doing which action when giving respiratory care to this client? 1. Encourage hourly coughing. 2. Assist with incentive spirometer. 3. Encourage hourly deep breathing. 4. Reposition gently side to side every 2 hours.

1. Encourage hourly coughing. Rationale: With aneurysm precautions, any activity that could raise the client's intracranial pressure (ICP) is avoided. For this reason, activities such as straining, coughing, blowing the nose, and even sneezing are avoided whenever possible. The other interventions (repositioning, deep breathing, and incentive spirometry) do not provide added risk of increasing ICP and are beneficial in reducing the respiratory complications of bed rest. Source: IGGY

A student nurse is assisting with an assessment of a client's level of consciousness using the Glasgow Coma Scale. The student understands that which categories of client functioning are included in this assessment? Select all that apply. 1. Eye opening 2. Reflex response 3. Best verbal response 4. Best motor response 5. Pupil size and reaction

1. Eye opening 3. Best verbal response 4. Best motor response Rationale: Assessment of pupil size and reaction and reflex response are not part of the Glasgow Coma Scale. The three categories included are eye opening, best verbal response, and best motor response. Pupil assessment and reflex response is a necessary part of a total assessment of the neurological status of a client but is not part of this particular scale. Source: IGGY

The nurse is caring for a client with an intracranial aneurysm who was previously alert. Which sign is an early indication that the level of consciousness (LOC) is deteriorating? Select all that apply. 1. Mild drowsiness 2. Drooping eyelids 3. Ptosis of the left eyelid 4. Slight slurring of speech 5. Less frequent spontaneous speech

1. Mild drowsiness 4. Slight slurring of speech 5. Less frequent spontaneous speech Rationale: Early changes in LOC relate to orientation, alertness and verbal responsiveness. Mild drowsiness, slight slurring of speech, and less frequent spontaneous speech are early signs of decreasing LOC. Ptosis (drooping) of the eyelid is caused by pressure on and dysfunction of cranial nerve III. Once ptosis occurs, it is ongoing; it does not relate to LOC.

The nurse has given the client with Bell's palsy instructions on preserving muscle tone in the face and preventing denervation. The nurse determines that the client needs additional teaching if the client makes which statements? 1. "I will perform facial exercises." 2. "I will expose my face to cold to decrease the pain." 3. "I will massage my face with a gentle upward motion." 4. "I will wrinkle my forehead, blow out my cheeks, and whistle frequently."

2. "I will expose my face to cold to decrease the pain." Rationale: Exposure to cold or drafts is avoided in Bell's palsy because it can cause discomfort. Prevention of muscle atrophy with Bell's palsy is accomplished with facial massage, facial exercises, and electrical nerves stimulation. Local application of heat to the face may improve blood flow and provide comfort.

The nurse is admitting a client to the hospital emergency department from a nursing home. The client is unconscious with an apparent frontal head injury. A medical diagnosis of epidural hematoma is suspected. Which question is of the highest priority for the emergency department nurse to ask of the transferring nurse at the nursing home? 1. "When did the injury occur?" 2. "Was the client awake and talking right after the injury?" 3. "What medications has the client received since the fall?" 4. "What was the client's level of consciousness before the injury?"

2. "Was the client awake and talking right after the injury?" Rationale: Epidural hematomas frequently are characterized by a "lucid interval" that lasts for minutes to hours, during which the client is awake and talking. After this lucid interval, signs and symptoms progress rapidly, with potentially catastrophic intracranial pressure increase. Epidural hematomas are medical emergencies. It is important for the nurse to assist in the differentiation between epidural hematoma and other types of head injuries.

A nurse is evaluating the neurological status of a client. To assess the function of the limbic system, the nurse should gather data about which item? 1. Experience of pain 2. Affect or emotions 3. Response to verbal stimuli 4. Insight, judgment, and planning

2. Affect or emotions Rationale: Affect and emotions are part of the role of the limbic system and involve both hemispheres of the brain. Source: IGGY

A client with myasthenia gravis is having difficulty with airway clearance and difficulty with maintaining an effective breathing pattern. The nurse should keep which items available at the client's bedside? 1. Oxygen and metered-dose inhaler 2. Ambu bag and suction equipment 3. Pulse oximeter and cardiac monitor 4. Incentive spirometer and cough pillow

2. Ambu bag and suction equipment Rationale: The client with myasthenia gravis may experience episodes of respiratory distress if excessively fatigued or with development of myasthenic or cholinergic crisis. For this reason, an Ambu bag, intubation tray, and suction equipment should be available at the bedside. Source: IGGY

The health care provider is preparing to administer edrophonium (Enlon) to the client with myasthenia gravis. In planning care, the nurse understands which about the administration of edrophonium? Select all that apply. 1. Edrophonium is a long-acting cholinesterase inhibitor. 2. Atropine is used to reverse the effects of edrophonium. 3. If symptoms worsen following administration of edrophonium, the crisis is cholinergic. 4. Edrophonium is used to distinguish between a myasthenic crisis and a cholinergic crisis. 5. An improvement in symptoms following administration of edrophonium indicates worsening of myasthenia gravis.

2. Atropine is used to reverse the effects of edrophonium. 3. If symptoms worsen following administration of edrophonium, the crisis is cholinergic. 4. Edrophonium is used to distinguish between a myasthenic crisis and a cholinergic crisis. 5. An improvement in symptoms following administration of edrophonium indicates worsening of myasthenia gravis.

The nurse is assisting in the care of a client with myasthenia gravis who is receiving pyridostigmine (Mestinon). Which medication should the nurse plan to have readily available should the client develop cholinergic crisis because of excessive medication dosage? 1. Vitamin K 2. Atropine sulfate 3. Protamine sulfate 4. Acetylcysteine (Mucomyst)

2. Atropine sulfate Rationale: If the client is in cholinergic crisis, the antidote for the medication would be a medication that is an anticholinergic. Thus, the antidote for cholinergic crisis is atropine sulfate. Vitamin K is the antidote for warfarin (Coumadin). Protamine sulfate is the antidote for heparin, and acetylcysteine (Mucomyst) is the antidote for acetaminophen (Tylenol).

A client who suffered a stroke is prepared for discharge from the hospital. The health care provider has prescribed range-of-motion (ROM) exercises for the client's right side. What nursing action should the nurse include in the client's plan of care? 1. Implement ROM exercises to the point of pain for the client. 2. Consider the use of active, passive, or active-assisted exercises in the home. 3. Encourage the client to be dependent on the home care nurse to complete the exercise program. 4. Develop a schedule of ROM exercises every 2 hours while awake even if the client is fatigued.

2. Consider the use of active, passive, or active-assisted exercises in the home. Rationale: The home care nurse must consider all forms of ROM for the client. Even a client with hemiplegia can participate in some components of rehabilitative care. In addition, the goal in home care nursing is for the client to assume as much self-care and independence as possible. The nurse needs to teach home care measures so that the client becomes self-reliant. Source: IGGY

A client is newly admitted to the hospital with a diagnosis of brain attack (stroke) manifested by complete hemiplegia. Which item in the medical history of the client should the nurse be most concerned? 1. Glaucoma 2. Emphysema 3. Hypertension 4. Diabetes mellitus

2. Emphysema Rationale: The nurse should be most concerned about emphysema. The respiratory system is the priority in the acute phase of a brain attack (stroke). The stroke client is vulnerable to respiratory complications such as atelectasis and pneumonia. Because the client has complete hemiplegia (is unable to move) and has emphysema, these risks are very significant. Although options 1, 3, and 4 are important, they are not as significant as option 2.

The nurse is caring for a client who is at risk for increased intracranial pressure (ICP) after a stroke. Which activities performed by the nurse will assist with preventing increases in ICP? Select all that apply. 1. Clustering nursing activities 2. Hyperoxygenating before suctioning 3. Maintaining 20 degree flexion of the knees 4. Maintaining the head and neck in midline position 5. Maintaining the head of the bed (HOB) at 30 degrees elevation

2. Hyperoxygenating before suctioning 4. Maintaining the head and neck in midline position 5. Maintaining the head of the bed (HOB) at 30 degrees elevation Rationale: Measures aimed at preventing increased ICP in the post-stroke client include hyperoxgenating before suctioning to avoid transient hypoxemia and resultant ICP elevation from dilation of cerebral arteries; maintaining the head in a midline, neutral position to help promote venous drainage from the brain; and keeping the HOB elevated to between 25 and 30 degrees to prevent a decreased blood flow to the brain. Clustering activities can be stressful for the client and increase ICP. Maintaining 20 degree flexion of the knees increases intra-abdominal pressure and consequently ICP. Source: IGGY

A nurse notes that a client who has suffered a brain injury has an adequate heart rate, blood pressure, fluid balance, and body temperature. The nurse concludes that which area of the client's brain is functioning adequately? 1. Thalamus 2. Hypothalamus 3. Limbic system 4. Reticular activating system

2. Hypothalamus Rationale: The hypothalamus is responsible for autonomic nervous system functions, such as heart rate, blood pressure, temperature, and fluid and electrolyte balance (among others). The thalamus acts as a relay station for sensory and motor information. The limbic system is responsible for emotions. The reticular activating system is responsible for the sleep-wake cycle. Source: IGGY

A client is admitted with an exacerbation of multiple sclerosis. The nurse is assessing the client for possible precipitating risk factors. Which factor, if reported by the client, should the nurse identify as being unrelated to the exacerbation? 1. Annual influenza vaccination 2. Ingestion of increased fruits and vegetables 3. An established routine of walking 2 miles each evening 4. A recent period of extreme outside ambient temperatures

2. Ingestion of increased fruits and vegetables Rationale: The onset or exacerbation of multiple sclerosis can be preceded by a number of different factors, including physical stress (e.g., vaccination, excessive exercise), emotional stress, fatigue, infection, physical injury, pregnancy, extremes in environmental temperature, and high humidity. No methods of primary prevention are known. Intake of fruits and vegetables is an unrelated item.

A client admitted to the nursing unit from the hospital emergency department has a C4 spinal cord injury. In conducting the admission assessment, what is the nurse's priority action? 1. Take the temperature. 2. Listen to breath sounds. 3. Observe for dyskinesias. 4. Assess extremity muscle strength.

2. Listen to breath sounds. Rationale: Because compromise of respiration is a leading cause of death in cervical cord injury, respiratory assessment is the highest priority. Assessment of temperature and strength can be done after adequate oxygenation is ensured. Because dyskinesias occur in cerebellar disorders, this is not as important a concern as in cord-injured clients unless head injury is suspected Source: IGGY

The nurse is performing an assessment on a client with Guillain-Barré syndrome. The nurse determines that which finding would be of most concern? 1. Difficulty articulating words 2. Lung vital capacity of 10 mL/kg 3. Paralysis progressing from the toes to the waist 4. A blood pressure (BP) decrease from 110/78 to 102/70 mm Hg

2. Lung vital capacity of 10 mL/kg Rationale: Respiratory compromise is a major concern in clients with Guillain-Barré syndrome. Clients often are intubated and mechanically ventilated when the vital capacity is less than 15 mL/kg. Options 1 and 3 are expected, depending on the degree of paralysis that occurs. Although orthostatic hypotension is a problem with these clients, the BP drop in option 4 is less than 10 mm Hg and is not significant.

The nurse is reviewing the record for a client seen in the health care clinic and notes that the health care provider has documented a diagnosis of amyotrophic lateral sclerosis (ALS). Which initial clinical manifestation of this disorder should the nurse expect to see documented in the record? 1. Muscle wasting 2. Mild clumsiness 3. Altered mentation 4. Diminished gag reflex

2. Mild clumsiness Rationale: The initial symptom of ALS is a mild clumsiness, usually noted in the distal portion of one extremity. The client may complain of tripping and drag one leg when the lower extremities are involved. Mentation and intellectual function usually are normal. Source: IGGY

A client was seen and treated in the hospital emergency department for treatment of a concussion. The nurse determines that the family needs reinforcement of the discharge instructions if they verbalize to call the health care provider (HCP) for which client sign or symptom? 1. Vomiting 2. Minor headache 3. Difficulty speaking 4. Difficulty awakening

2. Minor headache Rationale: A concussion after head injury is a temporary loss of consciousness (from a few seconds to a few minutes) without evidence of structural damage. After concussion, the family is taught to monitor the client and call the HCP or return the client to the emergency department for signs and symptoms such as confusion, difficulty awakening or speaking, one-sided weakness, vomiting, and severe headache. Minor headache is expected. Source: IGGY

A client with a neurological deficit is able, with eyes closed, to identify a set of keys placed in his or her hands. A nurse observing the client interprets this to mean that which area of the brain is intact? 1. Frontal lobe 2. Parietal lobe 3. Temporal lobe 4. Occipital lobe

2. Parietal lobe Rationale: The ability to distinguish an object by touch is called stereognosis, which is a function of the right parietal area. The parietal lobe of the brain is responsible for spatial orientation and awareness of sizes and shapes. The left parietal area is responsible for mathematics and right-left orientation. Source: IGGY

At the beginning of the work shift, the nurse assesses the status of the client wearing a halo device. The nurse determines that which assessment finding requires intervention? 1. Tightened screws 2. Red skin areas under the jacket 3. Clean and dry lamb's wool jacket lining 4. Finger-width space between the jacket and the skin

2. Red skin areas under the jacket Rationale: Red skin areas under the jacket indicate that the jacket is too tight. The resulting pressure could lead to altered skin integrity and needs to be relieved by loosening the jacket. The screws all should be properly tightened. A clean, dry lamb's wool lining should be in place underneath the jacket, and there should be a finger-width space between the jacket and the skin. In addition, the client should wear a clean white cotton T-shirt next to the skin to help prevent itching. Source: IGGY

The client with a cervical spine injury has cervical tongs applied in the emergency department. What should the nurse avoid when planning care for this client? 1. Using a Roto-Rest bed 2. Removing the weights to reposition the client 3. Assessment of the integrity of the weights and pulleys 4. Comparing the amount of prescribed traction with the amount in use

2. Removing the weights to reposition the client Rationale: Cervical tongs are applied after drilling holes in the client's skull under local anesthesia. Weights are attached to the tongs, which exert pulling pressure on the longitudinal axis of the cervical spine. Serial x-rays of the cervical spine are taken, with weights being added gradually until the x-ray reveals that the vertebral column is realigned. After that, weights may be reduced gradually to a point that maintains alignment. The client with cervical tongs is placed on a Stryker frame or Roto-Rest bed. The nurse ensures that weights hang freely, and the amount of weight matches the current prescription. The nurse also inspects the integrity and position of the ropes and pulleys. The nurse does not remove the weights to administer care.

A client has sustained damage to Wernicke's area in the temporal lobe from a brain attack (stroke). Which should the nurse anticipate when caring for this client? 1. The client will be unable to recall past events. 2. The client will have difficulty understanding language. 3. The client will demonstrate difficulty articulating words. 4. The client will have difficulty moving one side of the body.

2. The client will have difficulty understanding language. Rationale: Wernicke's area consists of a small group of cells in the temporal lobe the function of which is the understanding of language. The hippocampus is responsible for the storage of memory (the client will be unable to recall past events). Damage to Broca's area is responsible for aphasia (the client will demonstrate difficulty articulating words). The motor cortex in the precentral gyrus controls voluntary motor activity (the client will have difficulty moving one side of the body).

A client with recent-onset Bell's palsy is upset and crying about the change in facial appearance. The nurse plans to support the client emotionally by making which statement to the client? 1. This is caused by a small tumor, which can be removed easily. 2. This is not a brain attack (stroke), and many clients recover in 3 to 5 weeks. 3. This is a temporary problem, with treatment similar to that for migraine headaches. 4. This is similar to a brain attack (stroke), but all symptoms will reverse without treatment.

2. This is not a brain attack (stroke), and many clients recover in 3 to 5 weeks. Rationale: Clients with Bell's palsy should be reassured that they have not experienced a brain attack (stroke) and that symptoms often disappear spontaneously in 3 to 5 weeks. The client is given supportive treatment for symptoms. Bell's palsy usually is not caused by a tumor, and the treatment is not similar to that for migraine headaches.

A client who experienced a brain attack (stroke) several months ago still exhibits some difficulty with chewing food. The nurse plans care, knowing that the client has residual dysfunction of which cranial nerve? 1. Vagus (cranial nerve X) 2. Trigeminal (cranial nerve V) 3. Hypoglossal (cranial nerve XII) 4. Spinal accessory (cranial nerve XI)

2. Trigeminal (cranial nerve V) Rationale: The motor branch of cranial nerve V is responsible for the ability to chew food. The vagus nerve is active in parasympathetic functions of the autonomic nervous system. The hypoglossal nerve aids in swallowing. The spinal accessory nerve is responsible for shoulder movement, among other things. Source: IGGY

A nurse is assisting in the care of a client being discharged on phenytoin (Dilantin), 100 mg three times daily, for seizure control. When providing client teaching about this medication, the nurse should be sure to include which points? Select all that apply. 1. Break the capsules so they are easier to swallow. 2. Use a soft toothbrush while taking this medication. 3. If a dose is missed, just wait until the next one is due. 4. Alcohol should be avoided while taking this medication. 5. The medication may turn the client's urine pink, red, or brown. 6. Sore throat is a common side effect of the medication and is nothing to worry about.

2. Use a soft toothbrush while taking this medication. 4. Alcohol should be avoided while taking this medication. 5. The medication may turn the client's urine pink, red, or brown. Rationale: Phenytoin is an anticonvulsant that can cause gingival hyperplasia, as well as bleeding, swelling, and tenderness of the gums. The client should use good oral hygiene, gum massage, and have regular dental checkups. Alcohol interferes with the absorption of phenytoin, so it should be avoided. Change in the color of the urine is a normal reaction. A sore throat, fever, glandular swelling, or any skin reaction indicates hematological toxicity and needs to be reported. Source: Kee Hayes

The home care nurse is making extended follow-up visits to a client discharged from the hospital after a moderately severe head injury. The family states that the client is behaving differently than before the accident. The client is more fatigued and irritable and has some memory problems. The client, who was previously very even-tempered, is prone to outbursts of temper now. The nurse counsels the family on the basis of an understanding that these behaviors are indicative of which condition? 1. Indicate a worsening of the original injury 2. Will probably be a long-term sequela of the injury 3. Will come and go as intracranial pressure changes 4. Are short-term problems that will resolve in about 1 month

2. Will probably be a long-term sequela of the injury Rationale: Clients with moderate to severe head injury usually have residual physical and cognitive disabilities; these include personality changes, increased fatigue and irritability, mood alterations, and memory changes. The client also may require frequent to constant supervision. The nurse assesses the family's ability to cope and makes appropriate referrals to respite services, support groups, and state or local chapters of the National Head Injury Foundation. Source: IGGY

A nurse is testing the spinal reflexes of a client during neurological assessment. Which reflex will assist in determining that the client has an adequate spinal reflex? 1. Cough reflex 2. Withdrawal reflex 3. Accommodation reflex 4. Munroe-Kellie reflex

2. Withdrawal reflex Rationale: The withdrawal reflex is one of the spinal reflexes. It is an abrupt withdrawal of a body part in response to painful or injurious stimuli. The cough reflex is a brainstem-associated reflex. Accommodation reflex is associated with cranial nerve III and is part of the ocular motor system. Munroe-Kellie is not a reflex; it is a doctrine or a hypothesis addressing the cerebral volume relationships among the brain, the cerebrospinal fluid, and intracranial blood and their cumulative impact on intracranial pressure.

The home health nurse is visiting a client with a diagnosis of multiple sclerosis. The client has been taking oxybutynin (Ditropan XL). The nurse evaluates the effectiveness of the medication by asking the client which assessment question? 1. "Are you consistently fatigued?" 2. "Are you having muscle spasms?" 3. "Are you getting up at night to urinate?" 4. "Are you having normal bowel movements?"

3. "Are you getting up at night to urinate?" Rationale: Oxybutynin is an antispasmodic used to relieve symptoms of urinary urgency, frequency, nocturia, and incontinence in clients with uninhibited or reflex neurogenic bladder. Expected effects include improved urinary control and decreased urinary frequency, incontinence, and nocturia.

The nurse has instructed a client with myasthenia gravis about strategies for self-management at home. The nurse determines a need for more information if the client makes which statement? 1. "Here's the Medic-Alert bracelet I obtained." 2. "I should take my medications an hour before mealtime." 3. "Going to the beach will be a nice, relaxing form of activity." 4. "I've made arrangements to get a portable resuscitation bag and home suction equipment."

3. "Going to the beach will be a nice, relaxing form of activity." Rationale: Most ongoing treatment for myasthenia gravis is done in outpatient settings, and the client must be aware of the lifestyle changes needed to maintain independence. The client should carry medical identification about the presence of the condition. Taking medications an hour before mealtime gives greater muscle strength for chewing and is indicated. The client should have portable suction equipment and a portable resuscitation bag available in case of respiratory distress. The client should avoid situations and other factors, including stress, infection, heat, surgery, and alcohol, that could worsen the symptoms. Source: IGGY

The nurse has provided instructions to a client with a diagnosis of myasthenia gravis about home care measures. Which client statement indicates the need for further instruction? 1. "I will rest each afternoon after my walk." 2. "I should cough and deep breathe many times during the day." 3. "I can change the time of my medication on the mornings when I feel strong." 4. "If I get abdominal cramps and diarrhea, I should call my health care provider."

3. "I can change the time of my medication on the mornings when I feel strong." Rationale: The client with myasthenia gravis and his or her family should be taught information about the disease and its treatment. They should be aware of the side and adverse effects of anticholinesterase medications and corticosteroids and should be taught that timing of anticholinesterase medication is critical. It is important to instruct the client to administer the medication on time to maintain a chemical balance at the neuromuscular junction. If it is not given on time, the client may become too weak to even swallow. Resting after a walk, coughing and deep breathing many times over the day, and calling the health care provider when experiencing abdominal cramps and diarrhea indicate correct understanding of home care instructions to maintain health with this neurological degenerative disease. Source: IGGY

A home health nurse visits a client who suffered a back injury. On review of the health care provider's prescriptions, the nurse notes that codeine sulfate has been prescribed for the client, and the nurse provides instructions to the client regarding the medication. Which statement, if made by the client, indicates an understanding of health measures related to the medication? 1. "The medication is not addicting." 2. "I should watch out for diarrhea as a side effect." 3. "I should increase my fluid intake while taking this medication." 4. "I need to be sure to eat foods that are low in fiber to prevent diarrhea."

3. "I should increase my fluid intake while taking this medication." Rationale: Codeine sulfate is an opioid analgesic used to treat pain and can cause constipation. Because it is an opioid analgesic, codeine sulfate can be addicting. The client is instructed to increase fluid intake to prevent constipation. The client also should consume foods high in fiber and should take a stool softener.

The nurse is assisting in the care of a client who is being evaluated for possible myasthenia gravis. The health care provider gives a test dose of edrophonium (Enlon). Evaluation of the results indicates that the test is positive. Which would be the expected response noted by the nurse? 1. Joint pain for the next 15 minutes 2. An immediate increase in blood pressure 3. An increase in muscle strength within 1 to 3 minutes 4. Feelings of faintness or dizziness for 5 to 10 minutes

3. An increase in muscle strength within 1 to 3 minutes Rationale: Edrophonium (Enlon) is a short-acting acetylcholinesterase inhibitor used to diagnose myasthenia gravis. An increase in muscle strength should be seen in 1 to 3 minutes following the test dose if the client does have the disease. If no response occurs, another dose is given over the next 2 minutes and muscle strength is tested again. If no increase in muscle strength occurs with this higher dose, the muscle weakness is not caused by myasthenia gravis. Clients who receive injections of this medication commonly demonstrate a drop of blood pressure, feel faint and dizzy, and are flushed. Source: IGGY

A client has a prescription to receive valproic acid (Depakene) daily. To maximize the client's safety, when is the best time for the nurse to schedule the administration of this medication? 1. After lunch 2. After breakfast 3. Before bedtime 4. Before breakfast

3. Before bedtime Rationale: Valproic acid is an anticonvulsant that causes central nervous system (CNS) depression. For this reason, the side effects include sedation, dizziness, ataxia, and confusion. When the client is taking this medication as a single daily dose, administering it at bedtime negates the risk of injury from sedation and enhances client safety. The medication also should be administered at the same time each day.

A client who has had a brain attack (stroke) is being managed on the medical nursing unit. At 0800, the client was awake and alert with vital signs of temperature 98° F orally, pulse 80 beats/min, respirations 18 breaths/min, and blood pressure 138/80 mm Hg. At noon, the client is confused and only responsive to tactile stimuli, and vital signs are temperature 99° F orally, pulse 62 beats/min, respirations 20 breaths/min, and blood pressure 166/72 mm Hg. The nurse should take which action? 1. Reorient the client. 2. Retake the vital signs. 3. Call the health care provider (HCP). 4. Administer an antihypertensive PRN.

3. Call the health care provider (HCP). Rationale: The important nursing action is to call the HCP. The deterioration in neurological status, decreasing pulse, and increasing blood pressure with a widening pulse pressure all indicate that the client is experiencing increased intracranial pressure, which requires immediate treatment to prevent further complications and possible death. The nurse should retake the vital signs and reorient the client to surroundings. If the client's blood pressure falls within parameters for PRN antihypertensive medication, the medication also should be administered. However, options 1, 2, and 4 are secondary nursing actions. Source: IGGY

The nurse is preparing for the admission to the unit of a client with a diagnosis of seizures and is preparing to institute full seizure precautions. Which item is contraindicated for use if a seizure occurs? 1. Oxygen source 2. Suction machine 3. Padded tongue blade 4. Padding for the side rails

3. Padded tongue blade Rationale: Full seizure precautions include bed rest with padded side rails in a raised position, a suction machine at the bedside, having diazepam (Valium) or lorazepam (Ativan) available, and providing an oxygen source. Objects such as tongue blades are contraindicated and should never be placed in the client's mouth during a seizure. Source: IGGY

A nurse is transcribing a prescription for antibiotic therapy for a client with bacterial meningitis. The nurse understands that the medication that will be prescribed for the client will have which characteristic? 1. Metabolizes slowly 2. Acts within minutes to hours 3. Crosses the blood-brain barrier 4. Excretes readily and easily in the urine

3. Crosses the blood-brain barrier Rationale: A key consideration in prescribing medications that will affect the brain is the ability of the medication to cross the blood-brain barrier. If the medication cannot pass into the brain, it will not be effective. The duration of action, onset, and excretion (options 1, 2, and 4) apply to most medications and most uses but are not specific to those needed to treat meningitis.

The nurse is assessing a client with a brainstem injury. In addition to obtaining the client's vital signs and determining the Glasgow Coma Scale score, what priority intervention should the nurse plan to implement? 1. Check cranial nerve functioning. 2. Determine the cause of the accident. 3. Draw blood for arterial blood gas analysis. 4. Perform a pulmonary wedge pressure measurement.

3. Draw blood for arterial blood gas analysis. Rationale: Assessment should be specific to the area of the brain involved. The respiratory center is located in the brainstem. Assessing the respiratory status is the priority for a client with a brainstem injury. Source: IGGY

The nurse has just admitted to the nursing unit a client with a basilar skull fracture who is at risk for increased intracranial pressure (ICP). Pending specific health care provider prescriptions, the nurse should avoid placing the client in which positions? 1. Head midline 2. Neck in neutral position 3. Flat, with head turned to the side 4. Head of bed elevated 30 to 45 degrees

3. Flat, with head turned to the side Rationale: The client who is at risk for or with increased ICP should be positioned so that the head is in a neutral, midline position. The nurse should avoid flexing or extending the client's neck or turning the head from side to side. The head of the bed should be raised to 30 to 45 degrees. Use of proper positions promotes venous drainage from the cranium to keep ICP down. Source: IGGY

The post-head injury client opens eyes to sound, has no verbal response, and localizes to painful stimuli when applied to each extremity. How should the nurse document the Glasgow Coma Scale (GCS) score? 1. GCS = 3 2. GCS = 6 3. GCS = 9 4. GCS = 11

3. GCS = 9 Rationale: The GCS is a method is assessing neurological status. The highest possible score in the GCS is 15. A score lower than 8 indicates that coma is present. Motor response points are as follows: Obeys a simple response = 6; Localizes painful stimuli = 5; Normal flexion (withdrawal) = 4; Abnormal flexion (decorticate posturing) = 3; Extensor response (decerebrate posturing) = 2; No motor response to pain = 1. Verbal response points are as follows: Oriented = 5; Confused conversation = 4; Inappropriate words = 3; Responds with incomprehensible sounds = 2; No verbal response = 1. Eye opening points are as follows: Spontaneous = 4; In response to sound = 3; In response to pain = 2; No response, even to painful stimuli = 1. Using the GCS, a score of 3 is given when the client opens the eyes to sound. Localization to pain is scored as 5. When there is no verbal response the score is a 1. The total score is then equal to 9. Source: IGGY

A client with multiple sclerosis is experiencing muscle weakness, spasticity, and an ataxic gait. On the basis of this information, the nurse should include which client problem in the plan of care? 1. Inability to care for self 2. Interruption in skin integrity 3. Interruption in physical mobility 4. Inability to perform daily activities

3. Interruption in physical mobility Rationale: Multiple sclerosis is a chronic, nonprogressive, noncontagious degenerative disease of the central nervous system characterized by demyelinization of the neurons. Interruption in physical mobility is most appropriate for the client with multiple sclerosis experiencing muscle weakness, spasticity, and ataxic gait. Source: IGGY

Sumatriptan (Imitrex) is prescribed for a client. Based in this prescription, the nurse suspects that the client has which condition? 1. Sinus headache 2. Simple headache 3. Migraine headache 4. Episodic headache

3. Migraine headache Rationale: Sumatriptan (Triptan Combination) is used to treat migraine headaches. This medication constricts blood vessels around the brain and reduces substances in the body that can trigger headache pain. Sinus, simple, and episodic headaches are not treated with this medication.

A client with a history of myasthenic gravis presents at a clinic with bilateral ptosis and is drooling, and myasthenia crisis is suspected. The nurse assesses the client for which precipitating factor? 1. Getting too little exercise 2. Taking excess medication 3. Omitting doses of medication 4. Increasing intake of fatty foods

3. Omitting doses of medication Rationale: Myasthenic crisis often is caused by undermedication and responds to the administration of cholinergic medications. Cholinergic crisis (the opposite problem) is caused by excess medication and responds to withholding of medications. Too little exercise and excessive fatty food intake are incorrect. Overexertion and overeating possibly could trigger myasthenic crisis. Source: IGGY

The client with a spinal cord injury at the level of T4 is experiencing a severe throbbing headache with a blood pressure of 180/100 mm Hg. What is the priority nursing intervention? 1. Notify the health care provider. 2. Loosen tight clothing on the client. 3. Place the client in a sitting position. 4. Check the urinary catheter tubing for kinks or obstruction.

3. Place the client in a sitting position. Rationale: The client is demonstrating clinical manifestations of autonomic dysreflexia, which is a neurological emergency. The first priority is to place the client in a sitting position to prevent hypertensive stroke. Options 2 and 4 can then be done, and option 1 can be completed once initial interventions are done. Source: IGGY

A client with myasthenia gravis has difficulty chewing and has received a prescription for pyridostigmine (Mestinon). The nurse should check to see that the client takes the medication at what time? 1. With meals 2. Between meals 3. Just after meals 4. 30 minutes before meals

4. 30 minutes before meals Rationale: Pyridostigmine is a cholinergic medication used to increase muscle strength in the client with myasthenia gravis. For the client who has difficulty chewing, the medication should be administered 30 minutes before meals to enhance the client's ability to eat.

The nurse is preparing to care for a client who had a supratentorial craniotomy. The nurse should plan to place the client in which position? 1. Prone 2. Supine 3. Side-lying 4. Semi-Fowler's

4. Semi-Fowler's Rationale: After supratentorial surgery (surgery above the tentorium of the brain), the head of the client's bed usually is elevated 30 degrees to promote venous outflow through the jugular veins. Options 1, 2, and 3 denote incorrect positions after this surgery and these positions could result in edema at the surgical site and increased intracranial pressure. Source: IGGY

The nurse is planning care for the client with a neurogenic bladder caused by multiple sclerosis. The nurse plans for fluid administration of at least 2000 mL/day. Which plan would be most helpful to this client? 1. 400 to 500 mL with each meal and 500 to 600 mL in the evening before bedtime 2. 400 to 500 mL with each meal and additional fluids in the morning but not after midday 3. 400 to 500 mL with each meal, with all extra fluid concentrated in the afternoon and evening 4. 400 to 500 mL with each meal and 200 to 250 mL at midmorning, midafternoon, and late afternoon

4. 400 to 500 mL with each meal and 200 to 250 mL at midmorning, midafternoon, and late afternoon Rationale: Spacing fluid intake over the day helps the client with a neurogenic bladder to establish regular times for successful voiding. Omitting intake after the evening meal minimizes incontinence or the need to empty the bladder during the night. Source: IGGY

A client with myasthenia gravis becomes increasingly weaker. The health care provider (HCP) injects a dose of edrophonium (Enlon) to determine whether the client is experiencing a myasthenic crisis or a cholinergic crisis. The nurse expects that the client will have which reaction if in cholinergic crisis? 1. No change in the condition 2. Complaints of muscle spasms 3. An improvement of the weakness 4. A temporary worsening of the condition

4. A temporary worsening of the condition Rationale: An edrophonium injection makes the client in cholinergic crisis temporarily worse. An improvement of the condition indicates myasthenic crisis. The other two options are unrelated to the test. Source: IGGY

The nurse is preparing a plan of care for a client with a brain attack (stroke) who has global aphasia. The nurse should incorporate communication strategies into the plan of care because the client's speech will be characteristic of which finding? 1. Intact 2. Rambling 3. Characterized by literal paraphasia 4. Associated with poor comprehension

4. Associated with poor comprehension Rationale: Global aphasia is a condition in which the affected person has few language skills as a result of extensive damage to the left hemisphere. The speech is nonfluent and is associated with poor comprehension and limited ability to name objects or repeat words. The client with conduction aphasia has difficulty repeating words spoken by another, and speech is characterized by literal paraphasia with intact comprehension. The client with Wernicke's aphasia may exhibit a rambling type of speech. Source: IGGY

A client who has a spinal cord injury that resulted in paraplegia experiences a sudden onset of severe headache and nausea. The client is diaphoretic with piloerection and has flushing of the skin. The client's systolic blood pressure (BP) is 210 mm Hg. What should the nurse immediately suspect? 1. Return of spinal shock 2. Malignant hypertension 3. Impending brain attack (stroke) 4. Autonomic dysreflexia (hyperreflexia)

4. Autonomic dysreflexia (hyperreflexia) Rationale: Autonomic dysreflexia (hyperreflexia) results from sudden strong discharge of the sympathetic nervous system in response to a noxious stimulus. Signs and symptoms include pounding headache, nausea, nasal stuffiness, flushed skin, piloerection, and diaphoresis. Severe hypertension can occur, with a systolic BP rising potentially as high as 300 mm Hg. It often is triggered by thermal or mechanical events such as a kinking of catheter tubing, constipation, urinary tract infection, or any variety of cutaneous stimuli. The nurse must recognize this situation immediately and take corrective action to remove the stimulus. If untreated, this medical emergency could result in stroke, status epilepticus, or possibly death. Source: IGGY

The nurse is providing instructions to the client with trigeminal neuralgia regarding measures to take to prevent the episodes of pain. Which should the nurse instruct the client to do? 1. Prevent stressful situations. 2. Avoid activities that may cause fatigue. 3. Avoid contact with people with an infection. 4. Avoid activities that may cause pressure near the face.

4. Avoid activities that may cause pressure near the face. Rationale: The pain that accompanies trigeminal neuralgia is triggered by stimulation of the trigeminal nerve. Symptoms can be triggered by pressure such as from washing the face, brushing the teeth, shaving, eating, or drinking. Symptoms also can be triggered by stimulation by a draft or cold air. Source: IGGY

A client with vascular headaches is taking ergotamine (Cafergot). The home health nurse should periodically assess him or her for which finding? 1. Hypotension 2. Constipation 3. Dependent edema 4. Cool, numb fingers and toes

4. Cool, numb fingers and toes Rationale: Ergotamine produces vasoconstriction by stimulating α-adrenergic receptors, which suppresses vascular headaches when the medication is given in the therapeutic dose range. The nurse periodically assesses for hypertension; cool, numb fingers and toes; muscle pain; and nausea and vomiting. This medication does not cause hypotension, constipation, or dependent edema.

The nurse is preparing for the admission of a client with a suspected diagnosis of Guillain-Barré syndrome. When the client arrives at the nursing unit, the nurse reviews the health care provider's documentation. The nurse expects to note documentation of which hallmark clinical manifestation of this syndrome? 1. Multifocal seizures 2. Altered level of consciousness 3. Abrupt onset of a fever and headache 4. Development of progressive muscle weakness

4. Development of progressive muscle weakness Rationale: A hallmark clinical manifestation of Guillain-Barré syndrome is progressive muscle weakness that develops rapidly. Seizures are not normally associated with this disorder. The client does not have symptoms such as a fever or headache. Cerebral function, level of consciousness, and pupillary responses are normal Source: IGGY

The nurse is caring for a client receiving morphine sulfate for pain. Because this medication has been prescribed for this client, which nursing action should be included in the plan of care? 1. Encourage fluids. 2. Monitor the client's temperature. 3. Maintain the client in a supine position. 4. Encourage the client to cough and deep breathe.

4. Encourage the client to cough and deep breathe. Rationale: Morphine sulfate suppresses the cough reflex. Clients need to be encouraged to cough and deep breathe to prevent pneumonia. The remaining options are not associated specifically with the use of this medication. Source: Kee Hayes

A client who is taking phenytoin (Dilantin) for a seizure disorder is being admitted to the hospital because of an increase in seizure activity. The client reports severe vomiting for the last 24 hours and inability to take phenytoin during that time. The nurse anticipates that the health care provider will most likely prescribe which medication? 1. Phenobarbital (Luminal) 2. Clonazepam (Klonopin) 3. Valproic acid (Depakene) 4. Fosphenytoin sodium (Cerebyx)

4. Fosphenytoin sodium (Cerebyx) Rationale: Fosphenytoin sodium is used for short-term parenteral (intravenous) infusion. A client who is not tolerating medications orally and has a seizure disorder would need an anticonvulsant administered by the parenteral route. Phenobarbital is an antiseizure medication that is given orally or parenterally. However, the medication of choice in this case would be fosphenytoin since its use if for short-term. Valproic acid and clonazepam usually are administered orally.

A nurse is preparing to administer a prescribed antibiotic to a client with bacterial meningitis. The nurse understands that the selection of an antibiotic to treat meningitis is based on which fact? 1. It has a long half-life. 2. It acts within minutes to hours. 3. It can be easily excreted in the urine. 4. It is able to cross the blood-brain barrier.

4. It is able to cross the blood-brain barrier. Rationale: A primary consideration regarding medications to treat bacterial meningitis is the ability of the medication to cross the blood-brain barrier. If the medication cannot cross, it will not be effective. The duration, onset, and excretion of the medication(options 1, 2, and 3) are also of general concern but apply to all medications and not specifically to those that are used to treat meningitis. Source: IGGY

A postoperative craniotomy client who sustained a severe head injury is admitted to the neurological unit. What important nursing intervention is necessary for this client? 1. Take and record vital signs every 4 to 8 hours. 2. Prophylactically hyperventilate during the first 20 hours. 3. Treat a central fever with the administration of antipyretic medications such as acetaminophen (Tylenol). 4. Keep the head of the bed elevated at least 30 degrees, and position the client to avoid extreme flexion or extension of the neck and head.

4. Keep the head of the bed elevated at least 30 degrees, and position the client to avoid extreme flexion or extension of the neck and head. Rationale: Avoiding extreme flexion and extension of the neck can enhance venous drainage and help prevent increased intracranial pressure. As a general rule, hyperventilation is avoided during the first 20 hours postoperatively because it may produce ischemia caused by cerebral vasoconstriction. Vital signs need to be taken and recorded at least every 1 to 2 hours. Central fevers caused by hypothalamic damage respond better to cooling (hypothermia blankets, sponge baths) than to the administration of antipyretic medications. Source: IGGY

A nurse is speaking with a client taking phenytoin (Dilantin) for seizure control. The client states that she has started using birth control pills to prevent pregnancy. Which would be an important point for the nurse to emphasize to the client? 1. Oral contraceptives decrease the effectiveness of phenytoin. 2. Severe gastrointestinal side effects can occur when phenytoin and oral contraceptives are taken together. 3. There is an increased risk of thrombophlebitis when phenytoin and oral contraceptives are taken at the same time. 4. Phenytoin may decrease effectiveness of birth control pills, and additional measures should be taken to avoid pregnancy.

4. Phenytoin may decrease effectiveness of birth control pills, and additional measures should be taken to avoid pregnancy. Rationale: Phenytoin enhances the rate of estrogen metabolism, which can decrease the effectiveness of some birth control pills. The nurse should tell the client to alert the health care provider about the use of birth control pills so that counseling may be provided about alternative birth control methods. The other options are incorrect. Source: Kee Hayes

The nurse is caring for a client who is brought to the hospital emergency department with a spinal cord injury. The nurse minimizes the risk of compounding the injury most effectively by performing which action? 1. Keeping the client on a stretcher 2. Logrolling the client onto a soft mattress 3. Logrolling the client onto a firm mattress 4. Placing the client on a bed that provides spinal immobilization

4. Placing the client on a bed that provides spinal immobilization Rationale: Spinal immobilization is necessary after spinal cord injury to prevent further damage and insult to the spinal cord. Whenever possible, the client is placed on a special bed, such as a Stryker frame, which allows the nurse to turn the client to prevent complications of immobility while maintaining alignment of the spine. If a Stryker frame is not available, a firm mattress with a bed board under it should be used Source: IGGY

The nurse has a prescription to begin aneurysm precautions for a client with a subarachnoid hemorrhage secondary to aneurysm rupture. The nurse would plan to incorporate which intervention in controlling the environment for this client? 1. Keep the window blinds open. 2. Turn on a small spotlight above the client's head. 3. Make sure the door to the room is open at all times. 4. Prohibit or limit the use of a radio or television and reading.

4. Prohibit or limit the use of a radio or television and reading. Rationale: Environmental stimuli are kept to a minimum with subarachnoid precautions to prevent or minimize increases in intracranial pressure. For this reason, lighting is reduced by closing window blinds and keeping the door to the client's room shut. Overhead lighting also is avoided for the same reason. The nurse prohibits television, radio, and reading unless this is so stressful for the client that it would be counterproductive. In that instance, minimal amounts of stimuli by these means are allowed with approval of the health care provider. Source: IGGY

The client has an impairment of cranial nerve II. Specific to this impairment, what should the nurse should plan to do to ensure client safety? 1. Speak loudly to the client. 2. Test the temperature of the shower water. 3. Check the temperature of the food on the dietary tray. 4. Provide a clear path for ambulation without obstacles.

4. Provide a clear path for ambulation without obstacles. Rationale: Cranial nerve II is the optic nerve, which governs vision. The nurse can provide safety for the visually impaired client by clearing the path of obstacles when ambulating. Speaking loudly may help overcome a deficit of cranial nerve VIII (vestibulocochlear). Testing the shower water temperature would be useful if there were an impairment of peripheral nerves. Cranial nerve VII (facial) and IX (glossopharyngeal) control taste from the anterior two thirds and posterior third of the tongue, respectively

A client had a transsphenoidal resection of the pituitary gland. The nurse notes drainage on the nasal dressing. Suspecting cerebrospinal fluid (CSF) leakage, the nurse should look for drainage that is of which characteristic? 1. Serosanguineous only 2. Bloody with very small clots 3. Sanguineous only with no clot formation 4. Serosanguineous, surrounded by clear to straw-colored fluid

4. Serosanguineous, surrounded by clear to straw-colored fluid Rationale: CSF leakage after cranial surgery may be detected by noting drainage that is serosanguineous (from the surgery) and surrounded by an area of clear or straw-colored drainage. The typical appearance of CSF drainage is that of a "halo." The nurse also would further verify actual CSF drainage by testing the drainage for glucose, which would be positive. Source: IGGY

The nurse is planning care for a client with intracranial pressure (ICP) monitoring. Which intervention is appropriate to include in the plan of care? 1. Place the client in Sims position. 2. Change the drainage tubing every 48 hours. 3. Level the transducer at the lowest point of the ear. 4. Use strict aseptic technique when touching the monitoring system.

4. Use strict aseptic technique when touching the monitoring system. Rationale: Because there is a foreign body embedded in the client's brain, vigilant aseptic technique should be implemented. Sims is side-lying, flat position. With a client who has increased ICP, the head of the bed should be elevated at least 30% to improve jugular outflow. The drainage tubing should not be routinely changed. It should remain for the duration of the monitoring. To obtain accurate ICP pressure readings, the transducer is zeroed at the level of the foramen of Monro, which is approximated by placing the transducer 1 inch above the level of the ear. Serial ICP readings should be done with the client's head in the same position.

The nurse is caring for a client with trigeminal neuralgia (tic douloureux). The client asks for a snack and something to drink. The nurse should offer which best snack to the client? 1. Cocoa with honey and toast 2. Hot herbal tea with graham crackers 3. Iced coffee and peanut butter and crackers 4. Vanilla wafers and room-temperature water

4. Vanilla wafers and room-temperature water Rationale: Because mild tactile stimulation of the face can trigger pain in trigeminal neuralgia, the client needs to eat or drink lukewarm, nutritious foods that are soft and easy to chew. Extremes of temperature will cause trigeminal nerve pain. Source: IGGY


Kaugnay na mga set ng pag-aaral

Quadratic Transformation_Describe

View Set

Princeton Review MCAT Psych/Soc Glossary, MCAT Kaplan Psychology/Sociology Vocabulary, MCAT Psychology and Sociology (Kaplan)

View Set

Manufacturing Processes Chapter 6

View Set

Geography Review Questions for Feb. 2016 Midterm 1st Section

View Set

Chapter #'s 13-16 & 21-22 PrepU Questions

View Set

MKT CH 5, MKT CH 6, MKT CH 7, MKT CH 8

View Set

Intro to Hospitality Management - Tourism (Chapter 8)

View Set